×

UPSC Courses

UPSC Previous Year Solved Papers

GEOGRAPHY

1. Match List I with List II and select the correct answer by using the codes given below the lists:

Explanation: C. Madras is warmer than Calcutta, because it is located towards the equator. Snowfall in Himalayas is due to altitude because increase in the altitude causes decrease in temperature. Rainfall decrease from Coastal regions to interior land regions, as the moisture laden winds loses most of their moisture in the regions nearer to sea. Winter rainfall in North India is due to Western depressions.

2. Match List I with List II and select the correct answer using the codes given below the lists:

Explanation: B. Deccan traps were formed in Cretaceous Eocene age. Western Ghats were formed in Late Cenozoic age. Aravallis were formed in pre-Cambrian age. Narmada-Tapi deposits were formed in Pleistocene age.

3. The January isotherm taken as a basis for dividing India into tropical and subtropical

Explanation: C. The January isotherm of 12

4. Match and select the correct answer by using the codes given below the lists:

Explanation: B. Coal mines are rich in Jharia, Girdih and Karanpura of Jharkhand. Gold mines are rich in Kolar and Hutti regions of Karnataka. Mica is available in Visakhapatnam, Nellore and Krishna districts of Andhra Pradesh. Manganese is available in Bhandara and Ratnagiri districts of Maharashtra.

5. The Alamatti Dam is on the river

Explanation: C. Alamatti dam is on the river Krishna.

6. Consider the map given below :

The predominant languages spoken in the areas marked A, B, C and D are respectively

Explanation: C. The area marked by A is Mathura region and there the Braj Bhasha is spoken. B is Lucknow region and Awadhi is spoken there. C and D region are near to North Bihar, there Bhojpuri and Maithili are spoken respectively.

7. The canal carrying capacity of Farakka is

Explanation: C. Farakka barrage is across river Ganges in West Bengal. Its carrying capacity is 75000 cusecs.

8. Which one of the following pairs is correctly matched?

Explanation: D. Sundari tree is found in Sundarbans of Bangladesh and India (Sunderban delta of West Bengal).

9. Which one of the following is cultivated by transplanting seedlings?

Explanation: C. Onion is cultivated by transplanting seedlings, whereas Maize, Sorghum and Soyabean are cultivated through direct seeding.

10. Consider the map given below?

The places marked A, B, C and D in the map are respectively

Explanation: C. The place marked as A is rift valley region of Narmada and Tapti rivers. B is Chattisgarh plain formed by Mahanadi river. C is Chhota Nagpur Plateau of Jharkhand. D is rain shadow region of western ghats.

11. Consider the map given below:

The places marked A, B, C and D in the map are respectively known for the cultivation of

Explanation: B. A is Gujarat region known for groundnut cultivation. B is known for sugarcane cultivation in Western Maharashtra. C is Odisha region for ragi cultivation. D is Andhra Pradesh region for Tobacco cultivation.

12. Which of the following places are known for paper manufacturing industry?
  1. Yamunanagar
  2. Guwahati
  3. Shahabad
  4. Ballarpur

Choose the correct answer using the codes given below:

Explanation: B. Yamunanagar, Guwahati and Ballarpur are in Haryana, Assam and Maharashtra respectively.

13. The tribal population in Andaman and Nicobar Islands belongs to the

Explanation: D. The tribes Onges and Jarwas of Andaman and Nicobar Islands belong to the Negroid race.

14. Match List I (Hazardous industries using child labour leading to the filing of a public interest petition in the Supreme Court) with List II (Located at) and select the correct answer by using the codes given below the lists:

Explanation: A. Glass Industry - Ferozabad ; Brassware

15. In the map shown in the given figure, rivers labelled as 1, 2, 3 and 4 are respectively

Explanation: A. The rivers marked in the map are: 1. Kosi, 2.Gomati, 3. Ghaghara, 4. Gandak.

16. About 50% of the world population is concentrated between the latitudes of

Explanation: B. Between 20

17. The tail of a comet is directed away from the sun because

Explanation: A. A comet is an icy body that releases gas or dust. They contain dust, ice, Carbon dioxide, ammonia, methane, etc. Comets orbit the sun but most are believed to inhabit in an area known as the East cloud. As the comet rotates around the sun, due to the centrifugal force, the lighter mass of the comet is away and appears as a tail. It is directed away from the sun.

18. The river shown on the map is

Explanation: D. The river shown in the map is Salween, it originates from plateau of Tibet and flows through Shan plateau in Eastern region of Myanmar.

19. Which one of the following factors is responsible for the change in the regular direction of the ocean currents in the Indian Ocean?

Explanation: B. Due to the Monsoon drift, ocean current in the Indian Ocean becomes Eastward and Westward in june and November respectively. It is affected by wind circulation. In summers North Equatorial ocean currents disappear and south west monsoon current are developed. Indian counter current also disappears in North Indian ocean due to monsoon drift.

20. Match List I with List II and select the correct answer by using the codes given below the lists:

Explanation: A. Barley is a temperate crop. It requires cooler climate. Rice requires high temperature and high rainfall and nutrient rich soil. Millets can be grown in less water and poor soil conditions. Tea can be grown in high altitudes with warm and moist climate.

21. "From Aceh in the far northwest to Torres Strait in the east is 5000 miles, almost as far as from London to Baghdad. The Archipelago has 14,000 islands, some mere equatorial rocks, others some of the largest in the world." This description best fits

Explanation: D. The description best fits to Indonesia as the country sprawled across, with 14,000 islands in the equatorial region and North-western tip of the country in Banda Aceh.

22. The group of small pieces of rock revolving round the sun between the orbits of Mars and Jupiter are called

Explanation: D. The group of small pieces of rocks revolving round the sun, between the orbits of Mars and Jupiter are called asteroids.

23. If the earth's direction of rotation is reversed, what would be the 1ST when it is noon at the International Date Line?

Explanation: A. Generally, IST is 5:30 hrs, ahead of the Greenwitch Mean Time. So if its noon at the International Date Line (Greenwich line) then IST is 12:00+5:30 = 17:30 hrs. If the Earths direction of rotation is reversed, the IST will behind Greenwich Mean Time. So if its noon at the International Dateline then IST will be 12:00-5:30=6:30hrs.

24. Which one of the following stars is nearest to the earth?

Explanation: C. Sun is nearest star of Earth, with its average distance being 149.6 million km and Proxima Centauri is the second nearest star of the Earth.

25. Which one of the following conditions is most relevant for the presence of life on Mars?

Explanation: C. The most relevant condition for the presence of life on Mars is occurrence of ice caps and frozen water. Because the same situations, on the Earth also, are supposed to support the primitive forms of life.

26. Consider the geographical details given in the following figure:

The point marked by A in the above figure indicates a country in

Explanation: C. As the longitudes 5

27. Consider the map given below:

Of the fourshaded areas in the map, that which is characterised by hot dry summers, mild and moist winters and seasonal reversal of winds is the area labeled

Explanation: A. The hot dry summers, mild and moist winters and seasonal reversal of winds is the characteristic of Mediterranean type of climate. Found between 30

28. Daily weather map showing isobars is an example of

Explanation: B. The third dimension is shown by series of lines called isopleths which connect points of equal value. Daily weather map showing isobars i.e. places having equal pressures is an example of Isopleth map.

29. Which one of the following countries is the largest producer of fuelwood in the world?

Explanation: B. Russia has the largest forest covers in the world.

30. One will NOT have to pass through the Suez Canal while going from Mumbai to

Explanation: B. Suez is the Eastern entry point of the Suez canal which is nearest to Bombay, where as port said is in the North western point of suez canal. Alexandria (Egypt), Benghazi (Libya) are away from Suez canal and in order to reach these ports, one has to travel through the Suez canal.

31. During a flight from Delhi to Tokyo the following are the landing airports:
  1. Hong Kong
  2. Hanoi
  3. Taipei
  4. Bangkok

The correct sequence of the landing at these airports during an onward journey is

Explanation: B. During flight to Delhi to Tokyo, the sequence of airports is Bangkok (Thailand), Hanoi (Vietnam),Hongkong, Taipei (Taiwan).

32. Which one of the following is present in the largest amount in terms of per cent by mass in the earth's crust?

Explanation: B. Oxygen (47%) in the form of oxides, is present in the largest amount second abundant is silicon (27%), third abundant is Aluminium (8%) in Earth's Crust.

33. Consider the following statements regarding asteroids:
  1. Asteroids are rocky debris of varying sizes orbiting the Sun.
  2. Most of the asteroids are small but some have diameter as large as 1000 km.
  3. The orbit of asteroids lies between the orbits of Jupiter and Saturn.

Of these statements:

Explanation: C. Asteroids are rocky debris of varying sizes from smaller to as large as 1000km. But the orbit of asteroids (tier orbit) lies between diameter of the Mars and Jupiter, but not between Jupiter and saturn. So, 1 and 2 are correct but not 3.

34. The correct sequence of different layers of the atmosphere from the surface of the Earth upwards is

Explanation: C. The correct sequence is Tropo-Strato-Meso-Ionospheres. Tropospheres extends upto 8-18km, after that, stratosphere upto 50 km, after that Mesosphere extends upto 80km and beyond that, upto 400 km Ionosphere exists.

35. Match rivers labelled A, B, C and D on the given map with their names given in the list and select the correct answer using the codes given below the list:

Codes:

Explanation: B. A is Yukon river in Alaska region. B is Mackenzie river in Northern Canada. C is St. Lawrence river in Eastern Canada. D is Orinoco river in Venezuela.

36. Estuaries possess distinct blooms of excessive growth of a pigmented dinoflagellates. These blooms are called

Explanation: A. The growth of pigmented dinoflagellates causes red colour to water. So these blooms are called Red tides. Algal bloom may also deplete oxygen in the waters and may release toxins that may be harmful to humans and other marine animals.

37. Consider the following climatic conditions (northern hemisphere):

These are most likely to be found in the natural regions of

Explanation: A. This is most likely to be found in the natural region of China type; which is typified by a warm moist summer and a cool, dry winter. Temperatures are strongly modified by maritime influence.

38. When there is noon at G. M .T. meridian people on another place of the Earth are taking their 6 o'clock morning tea. The longitude of the place is

Explanation: B. Since the earth makes one complete rotation of 360

39. Match List I with List II and select the correct answer by using the codes given below the lists:

Explanation: D. The smallest planet of the Solar system is Pluto. Largest planet of the solar system is Jupiter. Venus is the planet second from the sun in the solar system. Mercury is Planet nearest to the sun.

40. The satellites of which one of the following countries have helped in the preparation of a detailed and complete map of Antarctica?

Explanation: A. The satellites launched by Canada has helped in the preparation of a complete and high resolution map Antarctica.

41. Which one of the following countries has replaced Italy as the major importer of bauxite from India?

Explanation: A. Canada is largest importer of bauxite from India.

42. Indonesian forest fire in 1997 was caused by

Explanation: D. The Indonesian forest fire in 1997 was caused by Slash and burn technique of Agriculture.

43. The water pollution in river is measured by the dissolved amount of

Explanation: D. The water pollution in rivers is measured by the dissolved amount of Oxygen. Polluted water has high Biological Oxygen Demand (BOD), i.e., less amount of oxygen is dissolving in it.

44. What is the correct sequence of the following Indian states in descending order of their length of surface roads per 100 km2 of their area?
  1. Haryana
  2. Maharashtra
  3. Punjab
  4. Tamil Nadu

Select the correct answer using the codes given below:

Explanation: D. Punjab-Tamil Nadu- Maharashtra-Haryana.

45. The four railway junctions shown by numerals 1, 2, 3, 4 on the rough outline map of Gujarat are respectively

Explanation: B. The Railway junctions marked as 1, 2, 3, 4 are Mahasena, Surendranagar, Rajkot and Junagarh.

46. Which one of the following is the correct sequence of the states (labelled 1, 2, 3 and 4) of India shown on the map in descending order in terms of their available ground water resources for irrigation?

Explanation: D. Interma of available ground water resources for irrigation, the decreasing order is: Uttara Pradesh > Maharashtra > West Bengal > Assam.

47. Among the Indian states shown labelled 1, 2, 3 and 4 in the rough outline map given, the correct sequence of descending order of per cent of scheduled tribe population to their total population is:

Explanation: B. The descending order of given states, accprding to their percent of ST population to their total population is- Mizoram (94.75%) > Arunachal Pradesh (63.66%) > Manipur (34.41%) > Orissa (22.2%).

48. Which one of the following sets of states stands to benefit the most from the Konkan Railway?

Explanation: A. The states of Goa, Karnataka, Maharashtra, and Kerala would benefit most from the konkan railway, as it passes through these states.

49. Which one of the following languages belongs to the Austric group?

Explanation: C. Khasi and Nicobarese languages belongs to Monkhmer branch of Austric languages. Ladakhi belongs to Sino-Tibetan family. Marathi belongs to Indo Aryan family. Tamil belongs to Dravidian language family.

50. Which one of the following east flowing rivers of India has rift valley due to down warping?

Explanation: B. Mahanadi, the east flowing river of India has rift valley due to down warping.

51. Forest areas have been labelled as 1, 2, 3 and 4 in the rough outline map given:

Among these, those which were threatened in 1997 by a serious epidemic include

Explanation: C. The place marked 3 indicates sal trees of Madhya Pradesh. They were affected in 1997 by the insects.

52. Commercial production of mineral oil has started recently in which one of the areas of India, labelled 1, 2, 3 and 4 in the rough map given below:

Explanation: C. It is the Kaveri basin in Tamil Nadu. Note: Barmer in Rajasthan has been discovered to have new off shore commercial production in India.

53. Match List I with List II and select the correct answer using the codes given below the lists:

Explanation: B. Lead ore found in Zawar in Udaipur district in Rajasthan. Silver ore found in Bellary in Karnataka, baramulla in Jammu and Kashmir, Almora in Uttara Pradesh, Cuddapah, Guntur Kurnool in Andhra Pradesh. Graphite found in Rampa Chodavaram, Eastern Godavari District in Andhra Pradesh. Salt lakes of Sambhar in Jaipur, Phalodi and Pachabhadra in Jodhpur of Rajasthan.

54. The discovery of Oak flora in 1966 added a new chapter to the history of Indian Sericulture. Which one of the following states is the leading producer of Oak tasar silk?

Explanation: C. Mainly four varieties of silk are produced in India, i.e., Mulbery, Eri, Tasar and Muga. 80% of the silk produced in the Country is of mulberry silk mainly produced in Karnataka, Andhra Pradesh and Tamil Nadu. Muga is having a golden sheen and is found in Assam and North-Eastern states. Eri is also grown in Assam and North Eastern states. Tropical tasar is found in central India in Jharkhand, Chattisgarh, Bihar, M.p, U.P etc.

55. Match List I with List II and select the correct answer using the codes given below the lists:

Explanation: B. Jamnagar is important centre of woolen industry. Korba is important for Aluminium industry. Hospet in Bellary district is important for Iron and steel (The Vijaynagar steel plant). Haldia is for Fertilisers.

56. In the vicinity of Mumbai, a number of specialised towns have been developed. Match the lists of specialisation with towns and select the correct answer using the codes given below the lists:

Explanation: D. Alibag

57. Match List I with List II and select the correct answer using the codes given below the lists:

Explanation: A. Gram

58. The rough outline map given shows centres of cement industry labelled 1, 2, 3 and 4. Match these centres with the following sets of names:

Codes:

Explanation: B. Katni in Madhya Pradesh, Tirunelveli in Kerala, Sikka in Gujarat and Churk in Uttara Pradesh are the important centres of Cement Industry.

59. Which one of the following ports shown on the rough outline map of India is a riverine port?

Explanation: D. Hooghly in West Bengal is a major riverine port.

60. Match the following research institutes A, B, C and D with their respective location labelled as 1 to 6 in the given rough outline map:

Select the correct answer using the codes given below:

Explanation: A. Central Drug Research Institute is at Lucknow. National Atlas and Thematic Mapping Organisation is at Kolkata. National Institute of Ocean Technology is at Chennai. Temperate Forest Research Centre is at Shimla.

61. Some people in Manipur live in houses built on floating islands of weeds and decaying vegetation held together by suspended silt. These islands are called

Explanation: C. Phoomdis are a series of floating islands, exclusive to the Loktak lake in Manipur. They cover a substantial part of the lake area and are heterogenous masses of vegetation, soil and organic matter in different stages of decay.

62. Consider the following statements regarding asteroids:
  1. Asteroids are rocky debris of varying sizes orbiting the Sun
  2. Most of the asteroids are small but some have diameter as large as 1000 km.
  3. The orbit of asteroids lies between the orbits of Jupiter and Saturn.

Of these statements:

Explanation: C. Asteroids are rocky debris of varying sizes from smaller to as large as 1000km. But the orbit of asteroids (tier orbit) lies between diameter of the Mars and Jupiter, but not between Jupiter and saturn. So, 1 and 2 are correct but not 3.

63. The correct sequence of different layers of the atmosphere from the surface of the Earth upwards is

Explanation: C. The correct sequence is Tropo-Strato-Meso-Ionospheres. Tropospheres extends upto 8-18km, after that, stratosphere upto 50 km, after that Mesosphere extends upto 80km and beyond that, upto 400 km Ionosphere exists.

64. Match rivers labelled A, B, C and D on the given map with their names given in the list and select the correct answer using the codes given below the list:

Codes:

Explanation: B. A is Yukon river in Alaska region. B is Mackenzie river in Northern Canada. C is St. Lawrence river in Eastern Canada. D is Orinoco river in Venezuela.

65. Estuaries possess distinct blooms of excessive growth of a pigmented dinoflagellates. These blooms are called

Explanation: A. The growth of pigmented dinoflagellates causes red colour to water. So these blooms are called Red tides. Algal bloom may also deplete oxygen in the waters and may release toxins that may be harmful to humans and other marine animals.

66. Consider the following climatic conditions (northern hemisphere):

These are most likely to be found in the natural regions of

Explanation: A. This is most likely to be found in the natural region of China type; which is typified by a warm moist summer and a cool, dry winter. Temperatures are strongly modified by maritime influence.

67. When there is noon at G. M .T. meridian people on another place of the Earth are taking their 6 o'clock morning tea. The longitude of the place is

Explanation: B. Since the earth makes one complete rotation of 360

68. Match List I with List II and select the correct answer by using the codes given below the lists:

Explanation: D. The smallest planet of the Solar system is Pluto. Largest planet of the solar system is Jupiter. Venus is the planet second from the sun in the solar system. Mercury is Planet nearest to the sun.

69. The satellites of which one of the following countries have helped in the preparation of a detailed and complete map of Antarctica?

Explanation: A. The satellites launched by Canada has helped in the preparation of a complete and high resolution map Antarctica.

70. Which one of the following countries has replaced Italy as the major importer of bauxite from India?

Explanation: A. Canada is largest importer of bauxite from India.

71. Indonesian forest fire in 1997 was caused by

Explanation: D. The Indonesian forest fire in 1997 was caused by Slash and burn technique of Agriculture.

72. The water pollution in river is measured by the dissolved amount of

Explanation: D. The water pollution in rivers is measured by the dissolved amount of Oxygen. Polluted water has high Biological Oxygen Demand (BOD), i.e., less amount of oxygen is dissolving in it.

73. What is the correct sequence of the following Indian states in descending order of their length of surface roads per 100 km2 of their area?
  1. Haryana
  2. Maharashtra
  3. Punjab
  4. Tamil Nadu

Select the correct answer using the codes given below:

Explanation: D. Punjab-Tamil Nadu- Maharashtra-Haryana.

74. The four railway junctions shown by numerals 1, 2, 3, 4 on the rough outline map of Gujarat are respectively

Explanation: B. The Railway junctions marked as 1, 2, 3, 4 are Mahasena, Surendranagar, Rajkot and Junagarh.

75. Which one of the following is the correct sequence of the states (labelled 1, 2, 3 and 4) of India shown on the map in descending order in terms of their available ground water resources for irrigation?

Explanation: D. Interma of available ground water resources for irrigation, the decreasing order is: Uttara Pradesh > Maharashtra > West Bengal > Assam.

76. Among the Indian states shown labelled 1, 2, 3 and 4 in the rough outline map given, the correct sequence of descending order of per cent of scheduled tribe population to their total population is:

Explanation: B. The descending order of given states, accprding to their percent of ST population to their total population is- Mizoram (94.75%) > Arunachal Pradesh (63.66%) > Manipur (34.41%) > Orissa (22.2%).

77. Which one of the following sets of states stands to benefit the most from the Konkan Railway?

Explanation: A. The states of Goa, Karnataka, Maharashtra, and Kerala would benefit most from the konkan railway, as it passes through these states.

78. Which one of the following languages belongs to the Austric group?

Explanation: C. Khasi and Nicobarese languages belongs to Monkhmer branch of Austric languages. Ladakhi belongs to Sino-Tibetan family. Marathi belongs to Indo Aryan family. Tamil belongs to Dravidian language family.

79. Which one of the following east flowing rivers of India has rift valley due to down warping?

Explanation: B. Mahanadi, the east flowing river of India has rift valley due to down warping.

80. Forest areas have been labelled as 1, 2, 3 and 4 in the rough outline map given:

Among these, those which were threatened in 1997 by a serious epidemic include

Explanation: C. The place marked 3 indicates sal trees of Madhya Pradesh. They were affected in 1997 by the insects.

81. Commercial production of mineral oil has started recently in which one of the areas of India, labelled 1, 2, 3 and 4 in the rough map given below:

Explanation: C. It is the Kaveri basin in Tamil Nadu. Note: Barmer in Rajasthan has been discovered to have new off shore commercial production in India.

82. Match List I with List II and select the correct answer using the codes given below the lists:
  1. A. Graphite 1. Bellary
  2. B. Lead 2. Didwana
  3. C. Salt 3. Rampa
  4. D. Silver 4. Zawar

Codes:

Explanation: B. Lead ore found in Zawar in Udaipur district in Rajasthan. Silver ore found in Bellary in Karnataka, baramulla in Jammu and Kashmir, Almora in Uttara Pradesh, Cuddapah, Guntur Kurnool in Andhra Pradesh. Graphite found in Rampa Chodavaram, Eastern Godavari District in Andhra Pradesh. Salt lakes of Sambhar in Jaipur, Phalodi and Pachabhadra in Jodhpur of Rajasthan.

83. The discovery of Oak flora in 1966 added a new chapter to the history of Indian Sericulture. Which one of the following states is the leading producer of Oak tasar silk?

Explanation: C. Mainly four varieties of silk are produced in India, i.e., Mulbery, Eri, Tasar and Muga. 80% of the silk produced in the Country is of mulberry silk mainly produced in Karnataka, Andhra Pradesh and Tamil Nadu. Muga is having a golden sheen and is found in Assam and North-Eastern states. Eri is also grown in Assam and North Eastern states. Tropical tasar is found in central India in Jharkhand, Chattisgarh, Bihar, M.p, U.P etc.

84. Match List I with List II and select the correct answer using the codes given below the lists:

Explanation: B. Jamnagar is important centre of woolen industry. Korba is important for Aluminium industry. Hospet in Bellary district is important for Iron and steel (The Vijaynagar steel plant). Haldia is for Fertilisers.

85. In the vicinity of Mumbai, a number of specialised towns have been developed. Match the lists of specialisation with towns and select the correct answer using the codes given below the lists:

Explanation: D. Alibag

86. Match List I with List II and select the correct answer using the codes given below the lists:
  1. Cotton 1. Madhya Pradesh
  2. Gram 2. Gujarat
  3. Black pepper 3. West Bengal
  4. Pineapple 4. Kerala

Codes:

Explanation: A. Gram

87. The rough outline map given shows centres of cement industry labelled 1, 2, 3 and 4. Match these centres with the following sets of names:

Codes:

Explanation: B. Katni in Madhya Pradesh, Tirunelveli in Kerala, Sikka in Gujarat and Churk in Uttara Pradesh are the important centres of Cement Industry.

88. Which one of the following ports shown on the rough outline map of India is a riverine port?

Explanation: D. Hooghly in West Bengal is a major riverine port.

89. Match the following research institutes A, B, C and D with their respective location labelled as 1 to 6 in the given rough outline map:

Select the correct answer using the codes given below:

Explanation: A. Central Drug Research Institute is at Lucknow. National Atlas and Thematic Mapping Organisation is at Kolkata. National Institute of Ocean Technology is at Chennai. Temperate Forest Research Centre is at Shimla.

90. Central Drug Research Institute is at Lucknow. National Atlas and Thematic Mapping Organisation is at Kolkata. National Institute of Ocean Technology is at Chennai. Temperate Forest Research Centre is at Shimla.

Explanation: C. Phoomdis are a series of floating islands, exclusive to the Loktak lake in Manipur. They cover a substantial part of the lake area and are heterogenous masses of vegetation, soil and organic matter in different stages of decay.

91. Along which one of the following meridians did India experience the first light of the sunrise of the new millennium?

Explanation: D. The first light of the sunrise of new millennium in India experienced at kathal Island of Andaman and Nicobar, which is located at 92

92. Consider the following statements:
  1. Tides are of great help in navigation and fishing.
  2. High tide enables big ships to enter or leave the harbour safely.
  3. Tide prevents siltation in the harbours.
  4. Kandla and Diamond Harbour are tidal ports.

Which of these statements are correct?

Explanation: D. All the statements are correct. Tides are great help in navigation and fishing (as Tides affect the reproductive activity of fish).

93. Consider the following provinces of former Yugoslavia:
  1. Bosnia
  2. Croatia
  3. Slovenia
  4. Yugoslavia

The correct sequence of these provinces from the east to the west is

Explanation: A. The correct sequence from east to west is Yugoslavia, Bosnia, Slovenia and Croatia.

94. Which one of the following lakes forms an international boundary between Tanzania and Uganda?

Explanation: C. Lake Victoria acts as the International boundary between Tanzania and Uganda. It is the largest lake in Africa and 2nd widest fresh water body in the world.

95. Which one of the following statements is not true?

Explanation: A. Indira Gandhi canal originated fro Harike barrage at Sultanpur on Sutlej but Ghaggar is a tributary of river Saraswati, which ends in the Thar desert.

96. Match the locations of ports labelled as A, B, C and D in the given map with the names of those ports and select the correct answer using the codes given below the names of the ports: Names of Ports:
  1. Veraval
  2. Karwar
  3. Tuticorin
  4. Kakinada

Codes:

Explanation: B. In the given map A marked port is Veraval of Gujarat, B marked port is Karwar of Karnataka, C marked port is Tuticorin of Tamil Nadu and D marked port is Kakinada of Andhra Pradesh.

97. Which one of the following pairs of primitive tribes and places of their inhabitation is not correctly matched?

Explanation: D. Korba tribe is found in Chhotanagpur of Chhattisgarh and Jharkhand, whereas kodagu a district in Karnataka.

98. Assertion (A): The frequency of floods in North Indian plains has increased during the last couple of decades. Reason (R): There has been reduction in the depth of river valleys due to deposition of silt.

Explanation: A. Siltation is the process of deposition of silt on the river bed through rain water, by which the depth of the river river reduces. The flood water crosses the river embankment, by which flood occurs in most part of north India.

99. Assertion (A): Ganga Plain is the most denselypopulated part of India. Reason (R): Ganga is the most harnessed river of India.

Explanation: B. The ganga plain is most important plain of India. The gangetic plain covers more than a fourth of the country's total surface. The alluvial fertile soil is very suitable for cultivation of different cereals, pulses and nuts. So bulk of the population resides in this plain area.

100. Match the different ports of Union Territory of Pondicherry labelled as A, B, C and D in the given map with their respective names and select the correct answer using the codes given below the list of ports: List of Ports of Pondicherry:
  1. Karaikal
  2. Mahe
  3. Pondicherry
  4. Yanam

Codes:

Explanation: A. A- Mahe; B-Karaikal; C-Pondicherry; D- Yaman.

101. Assertion (A): In Australia, cattle rearing is done more for meat than for milk. Reason (R): Australians are traditionally non-vegetarians.

Explanation: B. Assertion is correct, but reason is not the correct explanation of assertion but the statement is true. In Australia farmers rear cattle for meat export and most of the Australian people are non-vegetarians.

102. The correct sequence in decreasing order of the four sugarcane producing States in India is

Explanation: B. Uttara Pradesh, Maharashtra, Tamil nadu and Andhra Pradesh is the correct descending order of Sugarcane producing states in India.

103. Consider the following statements about the megacities of India:
  1. Population of each megacity is more than 5 million.
  2. All the megacities are important sea ports.
  3. Megacities are either national or State capitals.

Which of these statements are correct?

Explanation: D. All the mega cities of India are not important sea ports. E.g: Delhi which is the country capital and a mega city is not located near the sea and is also not a port city.

104. Which one of the following ports of India handles the highest tonnage of import cargo?

Explanation: D. Visakhapatnam is situated in Eastern coast of India in Andhra Pradesh. It handles the highest tonnage of import cargo.

105. Match the cities labelled as 1, 2, 3, 4 and 5 in the given map with the names of the institutes located in these cities and select the correct answer using the codes given below the names of the institutes.

Explanation: B. Central marine fisheries Research Institute is located in Cochin which is marked 5 in the map. Central Sheep Breeding Farm is located in Hissar which is marked 2 in the map. National Dairy Research Institute is situated in Karnal which is marked 1 in the map and National Institute of Agricultural Extension Management is situated in Bangalore which is marked 4 in the map.

106. Consider the following statements:
  1. Maharashtra has the highest acreage under jawar in India.
  2. Gujarat is the largest producer of groundnut in India.
  3. Rajasthan has the largest area of cultivable wastelands in India.
  4. Andhra Pradesh has the highest per hectare yield of maize in India.

Which of these statements are correct?

Explanation: C. Maharashtra has the highest acreage under Jowar cultivation followed by Karnataka. Gujarat is the largest producer of groundnut in India with a share more than 50%. Rajasthan has the highest cultivable wasteland in India. Madhaya Pradesh has the highest per hectare yield of maize in India and the largest producer of maize.

107. Match List I with List II and select the correct answer using the codes given below the Lists:
  1. A. Atlas Cycle Company Ltd 1.Bangalore
  2. B. Bharat Earth Movers Ltd 2. Bhubaneswar
  3. C. Indian Farmers Fertilisers Cooperative Ltd. 3. Kalol
  4. D. National Aluminium Company Ltd. 4. Sonepat

Codes:

Explanation: D. Atlas cycle company limited is situated in Sonepat, Bharat Earth Movers Ltd is located in Bengaluru, Indian Farmers Fertilisers Cooperative Ltd. Is located in Kalol, National Aluminium Company Ltd. Is located in Bhubaneswar.

108. Which one of the following statements is true according to 1991 Census data?

Explanation: D. According to the census of 1991, Bihar has the lowest literacy rate i.e 37.49%.

109. Match List I with List II and select the correct answer using the codes given below the Lists:

Explanation: B. Central Institute of Higher Tibetan Studies is located in Varanasi; Indira Gandhi Institute of Development Research is located in Mumbai, National Institute of Mental Health and Neurosciences is located in Bangalore; Central Institute of English and Foreign Languages is located in Hyderabad.

110. Match (Power generation plant) with (Feed material) and select the correct answer using the codes given below the Lists:
  1. A. M/s Gowthami Solvents Oil Limited, Andhra Pradesh 1. Rice husk
  2. B. M/s KM. Sugar Mills, Uttar Pradesh 2. Slaughterhouse waste
  3. C. M/s Satia Paper Mills, Punjab 3. Distillery spent wash
  4. D. M/s Al Kabeer Exports Limited, Andhra Pradesh 4. Black liquor

Codes:

Explanation: C. M/s Gowthami Solvents Oil Limited, Andhra Pradesh uses Rice husk as feed material; M/s KM. Sugar Mills, Uttar Pradesh- Distillery spent wash; M/s Satia Paper Mills, Punjab- Black liquor as feed material; M/s Al Kabeer Exports Limited, Andhra Pradesh

111. Which one of the following weather conditions is indicated by a sudden fall in barometer reading?

Explanation: A. In stormy weather condition the pressure of atmosphere varies, which causes sudden fall in barometer reading.

112. Who amongst the following was the first to state that the earth was spherical?

Explanation: A. Aristotle in 340 B.C first stated that the earth was spherical in his book "On the Heaven".

113. If the stars are seen to rise perpendicular to the horizon by an observer, he is located on the:

Explanation: A. If the stars are seen to rise perpendicular to the horizon by an observer then the observer is at the equator because celestical equator is an imaginary circle around the sky directly above the earth's equator. It is always 90

114. Consider the following statements made about sedimentary rocks:
  1. Sedimentary rocks are formed at earth's surfaceby the hydrological system
  2. Formation of sedimentary rocks involves theweathering of preexisting rocks
  3. Sedimentary rocks contain fossils
  4. Sedimentary rocks typically occur in layers

Which of these statements are correct?

Explanation: D. All the statements are true about the sedimentary rock.

115. Volcanic eruptions do not occur in:

Explanation: A. The basaltic rock found in Baltic sea is transported by glacier.

116. Quartzite is metamorphosed from:

Explanation: C. Quartzite is a metamorphosed form of sandstone.

117. Identify the correct order of the process of soil-erosion from the following:

Explanation: A. The correct order of soil erosion process is splash erosion, sheet erosion, rill erosion and gully erosion. Splash occurs when the raindrop hits the surface; explosive impact soil into individual particles. Sheet erosion is the detachment of soil particles by rain drop and their removal down slope by water flowing overland as a sheet in definite channels. Rill erosion refers to the development of small ephemeral concentrated flow paths which function as both sedimentary source and sediment delivery systems for erosion on hill slopes. Sheet erosion removes the this top layer by raindrop kill or orion occurs in lines which are less than 30 cm deep when these rills are deeper than 30 forming channels are called as gully erosion. Gully erosion occurs when water flows in narrow channels after heavy rain and melting snow.

118. Temperature and rainfall of a meteorological stationare given below:

Identify the region having the above climatic patterns from amongst the following:

Explanation: A. Concentration of Rainfall in winter and dry, warm summers is the characteristic of Mediterranean or western margin climate.

119. Assertion (A): During the neap tides. The high tide is lower and the low tide is higher than usual. Reason (R): The neap tide, unlike the spring tide, occurs on the new moon instead of on the full moon.

Explanation: C. The sun, the earth and the moon come in the position of quadrature (form-a right angle) on seventh and eighth day of every fortnight of a month and thus the tide producing forces of the sun and the moon works in opposite direction, causing low tide. This is lower in height than the normal tide and is called neap tide. But the spring tide occurs during full moon and are called high tides.

120. A class of animals known as Marsupials is a characteristic feature of:

Explanation: B. Marsupials are a group of mammals commonly have pouches or pocket of skin, where the mother nurses their young. These marsupians are found in Australia. The common example of marsupians are Kangaroo and Koala.

121. The above map is the Union Territory of:

Explanation: C. The given map is the union territory of Dadra and Nagar Haveli in western India. The capital is at Silvasa and major language of this Union Territory is Gujarati.

122. Match List-I with List-II and select the correct answer using the codes given below the lists:
  1. A. Fohn 1. Argentina
  2. B. Sarmun 2. Kurdistan
  3. C. Santa Ana 3. Californi
  4. D. Zonda 4. Alps

Codes:

Explanation: B. Fohn is the warm and dry wind of Alps region. Sarmun is the local wind of Kuristan region, Santa Ana is the local wind of California and Zonda is the local wind of Argentina.

123. The given map shows four towns of Central Asian region marked as 1, 2, 3 and 4. Identify these from the following lists and select the correct answer using the codes given below: Towns:
  1. Bishkek
  2. Ashikhabad
  3. Tashkent
  4. Dushanbe

Codes:

Explanation: A. Ashikabad is marked as '1'; Tashkent is marked as '2'; Bishken is marked as '3'; Dushanbe is marked as '4' in the given map.

124. Consider the following statements regarding environment issues in India:
  1. Gulf of Mannar is one of the biosphere reserves
  2. The Ganga Action Plan, phase II has been merged with the National River Conservation Plan.
  3. The National Museum of Natural History at New Delhi imparts non-formal education in environment and conservation
  4. Environmental Information System (ENVIS) acts as a decentralized information network for environment information

Which of these statements are correct?

Explanation: B. All the statements given in the options are correct.

125. In the above map, the black marks show distribution of:

Explanation: D. In the given figure of India, Mica is found in all the states of India like Jharkhand, Orissa, Bihar, Rajasthan, Bombay, Karnataka, West Bengal and Andhra Pradesh.

126. In the shaded area of the above map, the mean temperature for the month of July varies between:

Explanation: C. In the given map, the shaded area indicates the tropical wet and dry region, where the mean temperature for the month of July varies between 27.5

127. Mekong Ganga Co-operation Project is:

Explanation: B. Mekong

128. Assertion (A): Anticyclonic conditions are formed in winter season when atmospheric pressure is high and air temperatures are low. Reason (R): Winter rainfall in northern India causes development of anticyclonic conditions with low temperatures.

Explanation: A. Both assertion and reasons are true and reason is correct explanation of assertion. In north India, a feeble high-pressure region develops, with light winds moving outwards from this area. Influenced by the relief, these winds blow through the Ganga valley from west and north west, which forms an anticyclonic condition in northern part of India.

129. Consider the following statements regarding power sector in India:
  1. The installed capacity of power generation is around 95000 MW
  2. Nuclear plants contribute nearly 15% of total power generation
  3. Hydroelectricity plants contribute nearly 40% of total power generation
  4. Thermal plants at present account for nearly 80% of total power generation

Which of the statements is/are correct?

Explanation: A. According to the data of 2000, the power generation is 95000 MW. Nuclear plants contribute 2% of total power generation. Hydroelectricity plants contribute 25% of total production and 73% is contributed by thermal power plants.

130. The approximate age of the Aravallis range is:

Explanation: A. The Aravallis range are the oldest mountain in India. The approximate age of the Aravallis is 670 million years.

131. The high density of population in Nile valley and Islands of Java is primarily due to:

Explanation: A. Due to availability of fertile alluvial soil intensive agricultural practices was possible in the Nile river and islands of Java. So, in this regions high density of population is found.

132. Assertion (A): The amount of moisture in the atmosphere is related to latitude Reason (R): The capacity to hold moisture in the form of water vapour is related to temperature.

Explanation: A. The amount of moisture in atmosphere is related to latitude because the temperature depends upon the latitude and capacity to hold moisture is related to temperature.

133. Assertion (A): Unlike temperate forests, the tropical rain forests, if cleared, can yield productive farmland that can support intensive agriculture for several years even without chemical fertilizers. Reason (R): The primary productivity of the tropical rain forest is very high when compared to that of temperate forests.

Explanation: D. A is false but R is true.

134. Assertion (A): Areas lying within five to eight degrees latitude on either side of the equator receive rainfall throughout the year. Reason (R): High temperatures and high humidity cause convectional rain to fall mostly in the afternoons near the equator.

Explanation: A. The highest rainfall totals occur near the equator in the tropics, where the strong heating by the sun creates significant vertical uplifts of air and the formation of prolonged heavy showers and frequent thunderstorms.

135. Which one among the following covers the highest percentage of forest area in the world?

Explanation: A. Temperature coniferous forests covers the highest percentage of forest area in the world.

136. The waterfall 'Victoria' is associated with the river:

Explanation: D. Victoria waterfalls is associated with the river Zambezi which is situated in Africa.

137. Life expectancy is the highest in the world in:

Explanation: C. Life expectancy in Canada is 79.7 years, Germany - 77.8 years, Japan - 80.9, Norway - 78.9 years. Japan has the highest life expectancy in the world.

138. Consider the following statements:
  1. In Macedonia, ethnic Albanians are in a minority
  2. In Kosovo, Serbians are in a majority

Which of these statements is/are correct?

Explanation: A. In Macedonia, ethinic Albanian population is 23%, which is a good number in Macedonia. But in Kosovo 92% are Albanians and Serbians are in a minority.

139. Israel has common borders with:

Explanation: A. Israel has common borders with Lebanon, Syria, Jordan and Egypt.

140. Which among the following countries has the largest population?

Explanation: A. Indonesia has the largest population in the given countries.

141. Which one of the following countries is land locked?

Explanation: A. Bolivia is a land locked country, in South America the surrounding countries are Peru, Chile, Argentina, Brazil and Paraguay.

142. Which one of the following countries does not border the Caspian Sea?

Explanation: A. Armenia is a former Soviet republic. It is located in the Mountainous Caucasus region between Asia and Europe. It is a land locked country.

143. What is the correct sequence of the rivers

Explanation: B. The correct sequence of the river in descending order of their lengths are Godavari (1465 km), Narmada (1312 km), Mahanadi (858 km) and Tapti (724 km).

144. Which amongst the following States has the highest population density as per Census-2001?

Explanation: D. As per the census 2001, West Bengal has the highest population density of 904, Kerala has 819, Madhya Pradesh has 196 and Uttar Pradesh has 689.

145. Nanda Devi peak forms a part of:

Explanation: B. Nanda Devi peak are a part of Kumaon Himalayas located in Chamoli district of Uttaranchal.

146. Among the following cities, which one has the highest altitude above mean sea level?

Explanation: A. Bangalore is the city which is 937 metres above sea level, Delhi is 218 metres sea level, Jodhpur is 230 metres and Nagpur is 247.5 metres above the sea level.

147. Consider the following statements:
  1. Longitude of Jabalpur's location is between those of Indore and Bhopal.
  2. Latitude of Aurangabad's location is between those of Vadodara and Pune.
  3. Bangalore is situated more southward than Chennai

Which of these statements is/are correct?

Explanation: C. The longitude of Jabalpur's location is to the east of Bhopal, Bangalore is north of Channai.

148. Which one among the following states is smallest in area?

Explanation: D. The area of Andhra Pradesh is 160,205 sq km, area of Gujarat is 196,024 sq km, the area of Karnataka is 191, 791 sq km and Tamil Nadu is 130, 058 sq km.

149. Among the following cities, which one is nearest to the Tropic of Cancer?

Explanation: B. The Tropic of Cancer located at 23.5

150. Assertion (A): Eastern coast of India produces more rice than the western coast. Reason (R): Eastern coast receives more rainfall than the western coast.

Explanation: C. Assertion is true, but reason is false. The western coast receives more rainfall than eastern coast, but the temperature between 20-27

151. Consider the following statements:
  1. India is the original home of the cotton plant
  2. India is the first country in the world to develop hybrid cotton variety leading to increased production

Which of these statements is/are correct?

Explanation: C. India is the original home of cotton plant, and India ranks number one in the world accounting for 20% of the total area planted under cotton. The cotton hybrid variety H

152. The Genetic Engineering Approval Committee, whose permission is required for cultivation of any genetically modified crop such as Bt Cotton in India, is under the Union Ministry of:

Explanation: A. The genetic engineering approval committee comes under Ministry of Enviroment, Forest and Climate change.

153. Consider the following statements:
  1. India ranks first in the world in fruit production
  2. India ranks second in the world in export of tobacco

Which of these statements is/are correct?

Explanation: D. India ranked 2nd in the fruit production in the world and ranked 6th in production of tobacco.

154. Assertion (A): Thickness of the atmosphere is maximum over the Equator. Reason (R): High insolation and strong convection currents occur over the Equator.

Explanation: D. The circulation of atmosphere and oceans is funda-mentally caused by the fact that the amount of incoming solar radiation varies from a maximum at the equator to a minimum at the poles. More reflection and absorption of incoming radiation takes place in high latitude because of the greater thickness of atmosphere that must be penetrated. Thickness of the earth is maximum over the pole.

155. The great Asian river Mekong does not run through:

Explanation: B. Mekong is a major river in south eastern Asia. Mekong is known as the Lancang in China. It runs through Yunan province, China, forms the border between Myanmar and Laos and most of the border between Laos and Thailand and flows across Cambodia and Southern Vietnam before emptying to South China.

156. Latvia does not share its borders with which one of the following countries?

Explanation: D. Latvia does not share its border with Poland. Latvia only distinct border is the Battic Sea coast, which extends for 531 kilometers. Its neighbors include Lithuania on the south, Estonia on the north, Russia on the east and Belarus on the southeast.

157. Match List-I with List-II and select the correct answer using the codes given below:
  1. A. Black Sea 1. Bulgaria
  2. B. Red Sea 2. China
  3. C. Yellow Sea 3. Eritrea
  4. D. Caspian Sea 4. Kazakhstan

Codes:

Explanation: C. A-1; B-3; C-2; D-3.

158. Which one of the following does not border Panama?

Explanation: D. Panama is a country located in the central America region fo North America, bordering both the carribbean Sea and the North Pacific Ocean, between Colombia and costa Rica. Panama is located on the narrow and low Isthmus of Panama.

159. In which one of the following Union Territories, do people of the Onge tribe live?

Explanation: A. Onge is the major tribes of the union territory of Andaman and Nicobar. They were fully dependent on hunting and gathering. They belong to the Negrito race.

160. Consider the following statements:
  1. National Thermal Power Corporation has diversified into hydropower sector
  2. Power Grid Corporation of India has diversified into telecom sector

Which of the statements given above is/are correct?

Explanation: A. : Statement '1' is correct, because National Thermal Power Corporation has diversified into the hydro projects. Presently it has undertaken Koldem project in Himachal Pradesh.

161. Consider the following statements:
  1. Damodar, Valley Corporation is the first multi
  2. Damodar Valley Corporation includes thermal and gas power stations

Which of the statements given above is/are correct?

Explanation: A. Damodar Valley Corporation's (DVC) is first multi-purpose river valley project of independent. India was in the year 1954. The joint venture projects are Maithon Power Limited, Panchat and Tilayiya are hydroelectric power station. The DVC has under its operation four coal based thermal power stations, three hydel power stations and one gas turbine station so both the statement are correct.

162. Consider the following statements: Among the Indian States :
  1. Andhra Pradesh has the longest coastline
  2. Gujarat has the highest number of airports

Which of the statements given above is/are correct?

Explanation: B. Gujarat has longest coast line (1214.7 km) followed by Andhra Pradesh (973.7 km); Gujrat, with 10 airports, tops the list of states with the most number of operational airports.

163. Match List-I (Minerals) with List-II (Location) and select the correct answer using the codes given below:
  1. A. Coal 1. Giridih
  2. B. Copper 2. Jayamkondam
  3. C. Manganese 3. Alwar
  4. D. Lignite 4. Dharwar

Codes:

Explanation: C. A-1; B-3; C-4; D-2.

164. Which among the following National Highway routes is the longest?

Explanation: C. Length of Agra-Mumbai NH

165. Consider the following:
  1. Mahadeo Hills
  2. Sahyadri Parvat
  3. Satpura Range

What is the correct sequence of the above from the north to the south?

Explanation: C. Option C is correct answer.

166. Lake Sambhar is nearest to which one of the following cities of Rajasthan?

Explanation: B. Sambhar is India's largest salt lake which is located 96 km south west of city of Jaipur.

167. Match List I (National Park/Sanctuary) with List II (State) and select the correct answer using the codes given below:
  1. A. Kanger Ghati National Park 1. Chhattisgarh
  2. B. Nagerhole National Park 2. Haryana
  3. C. Kugti Wildlife Sanctuary 3. Himachal Pradesh
  4. D. Sultanpur Bird Sanctuary 4. Karnataka

Codes:

Explanation: B. A-1; B-4; C-1; D-2.

168. Which one of the following statements is not correct?

Explanation: A. Option A is not correct.

169. Assertion (A): Bangalore receives much higher average annual rainfall than that of Mangalore. Reason (R): Bangalore has the benefit of receiving rainfall both from south-west and north-east monsoons.

Explanation: D. Bangalore receives lesser rainfall than Mangalore. Mangalore is located in windward slope and receives 2000 mm of rainfall, whereas Bangalore present in rain shadow area receives less than 500 mm of water.

170. Assertion (A): West-flowing rivers of Peninsular India have no deltas. Reason (R): These rivers do not carry any alluvial sediments.

Explanation: A. West flowing rivers Narmada and Tapti do not form delta, because topography of western peninsular India is rocky without loose sediments and no alluvial sediments carried by the rivers.

171. Consider the following statements:
  1. Indira Gandhi Centre for Atomic Research uses fast reactor technology
  2. Atomic Minerals Directorate for Research and Exploration is engaged in heavy water production
  3. Indian Rare Earths Limited is engaged in manufacture of Zircon for India's Nuclear Programme beside other rare earth products

Which of the statements given above are correct?

Explanation: A. All the statements are correct according to Atomic developments in India.

172. Match List I (Beaches in India) with List II (States) and select the correct answer using the codes given below:

Explanation: B.

173. Match List-I (Biosphere Reserve) with List-II (States) and select the correct answer using the codes given below:

Explanation: D. Similipal biosphere reserve is in Orissa. Dehong-Debang biosphere reserve is in Arunachal Pradesh, Nokrek biosphere reserve is in Meghalaya and Kanchenjunga biosphere reserve is situated in Sikkim.

174. Amongst the following Indian States which one has the minimum total forest cover?

Explanation: C. Haryana has 6.83% of area is forest cover area of the total land coverage, whereas Sikkim has 36%, 38.5% in Goa and 28.9% of Kerala has forest area of their total land mass.

175. Consider the following crops:
  1. Cotton
  2. Groundnut
  3. Maize
  4. Mustard

Which of the above are kharif crops?

Explanation: B. Cotton, Groundnut and Maize are kharif crop whereas Mustard is a rabi crop.

176. Where is the volcanic mountain, Mount St. Helens located?

Explanation: B. Volcanic mountain. St. Helen is located in United States of America.

177. Assertion (A): The same face of the moon is always presented to the earth. Reason (R): The moon rotates about its own axis in 23 and half days which is about the same time that it takes to orbit the earth.

Explanation: C. The moon rotates about its own axis in 27.322 days. When the Moon first formed, its rotational speed and orbit were very different than they are now. Over time, the Earth's gravitational field gradually slowed the Moon's rotation until the orbital period and the rotational speed stabilized, making one side of the moon always face the Earth.

178. Assertion (A): Existence of human life on Venus is highly improbable. Reason (R): Venus has extremely high level of carbon dioxide in its atmosphere.

Explanation: A. The atmosphere of Venus is composed primarily of carbon dioxide and much denser and hotter than that of Earth. The atmosphere of Venus supports opaque clouds made of sulfuric acid, making optical Earth-based and orbital observation of the surface impossible.

179. Assertion (A): Wind patterns are clockwise in the Northern Hemisphere and anti-clockwise in the Southern Hemisphere. Reason (R): The directions of wind patterns in the Northern and the Southern Hemisphere are governed by the Coriolis effect.

Explanation: A. is true as the direction of wind in the northern hemisphere is clock wise and in southern hemisphere it is anti clock wise. The explanation is correct as this is mainly due to coriolis effect.

180. Where are the Balearic Islands located?

Explanation: A. The Balearic Islands are an archipelago of Spain in the Western Mediterranean sea, near the eastern coast of the Iberian peninsula.

181. Which one of the following pairs is not correctly matched?

Explanation: C. Managua is the capital of Nicaragua, not Balmopan. Balmopan is the capital city of Balize.

182. Consider the following statements:
  1. The Axis of the earth's magnetic field is inclined at 23 and half to the geographic axis of the earth.
  2. The earth's magnetic pole in the Northern Hemisphere is located on a Peninsula in Northern Canada.
  3. The earth's magnetic equator passes through Thumba in South India.

Which of the statements given above is/are correct?

Explanation: B. Earth magnetic field is inclined between

183. Which one of the following countries does not border Lithuania?

Explanation: B. Ukraine is a sovereign state in Eastern Europe, bordered by Russia to the east and north east; Belarus to the north west; Poland, Hungary and Slovakia to the west; Romania and Moldova to the south west; and the Black Sea and Sea of Agov to the South.

184. Which one of the following is the correct sequence of the given towns of Pakistan while moving from the north towards the south?

Explanation: C. While moving from the North to South Pakistan, the correct sequence of the towns is Peshawar

185. Match List-I with List-II and select the correct answer using the codes given below the lists:
  1. A. Washington, D.C. 1. River Manzanares
  2. B. Berlin 2. River Seine
  3. C. Paris 3. River Spree
  4. D. Madrid 4. River Potomac

Codes:

Explanation: D. Correct option is D.

186. Consider the following statements:
  1. Great Britain comprises England, Wales, Scotland and Northern Ireland.
  2. England covers less than 60% of the total area of the United Kingdom. .

Which of the statements given above is/are correct?

Explanation: B. The total area of United Kingdom is 94, 226 sq miles (244, 044 sq km), where England covers 50, 344 sq miles (130, 365 sq km) i.e. less than 60 % of total land area of United Kingdom. Great Britain donot comprise of Northern Ireland.

187. Itaipu Dam built on the river Parana is one of the largest dams in the world. Which of the following two countries have this as a joint project?

Explanation: C. Itaipu Dam is built on River Parana is the largest dam in the world. This is a joint project of Brazil

188. Which one of the following is the correct sequence of the given continents in the decreasing order of their percentage of Earth's land?

Explanation: B. The decreasing order of arrangement of continent according to the percentage of Earth's land is Africa (29800000 sq km), North America (21510000 sq km) South America (17598000 sq km) and Europe (9699550 sq km).

189. Consider the following statements:
  1. The forest cover in India constitutes around 20% of its geographical area. Out of the total forest cover, dense forest constitutes around 40%.
  2. The National Forestry Action Programme aims at bringing one third of the area of India under tree forest cover.

Which of the statements given above is/are correct?

Explanation: B. Out of the total forest cover, dense forest constitute around 2.54%; very dense forest and 8.77% are moderately dense forest. National Forestry Action Programme aims a long term plan to achieve the target of 33% forest cover.

190. Consider the following statements:
  1. Area-wise, Chhattisgarh is larger than West Bengal.
  2. According to the population Census 2001, population of West Bengal is larger than that of Chhattisgarh

Which of the statements given above is/are correct?

Explanation: C. Area of Chhattisgarh is 1,36,03489 km and Area of West Bengal is 88,572 sq km but the population density in Chhattigarh is 154, whereas the population density in West Bengal is 904.

191. Match List-I with List-II and select the correct answer using the code given below the lists.

Explanation: D. A-3; B-1; C-2; D-5.

192. Which one of the following statements is not correct?

Explanation: B. Bulk of the tribal population is found in 5 states, i.e., Madhaya Pradesh, Maharashtra, Orissa, Gujarat and Bihar.

193. Consider the following statements:
  1. India is the second country in the world to adopt a National Family Planning Programme.
  2. The National Population Policy of India 2000 seeks to achieve replacement level of fertility by 2010 with a population of 111 crores.
  3. Kerala is the first state in India to achieve replacement level of fertility.

Which of the statements given above is/are correct?

Explanation: C. Family Planning programme was started in 1952 by Government of India. Kerala is the first state in India to achieve replacement level of fertility.

194. Which one of the following statements is true on the basis of Census - 2001?

Explanation: D. Pondicherry, a Union Territory, has the sex ratio of 1001. Chandigarh has 777, Delhi has 861, Daman and Diu has 710, Dadra and Nagar Haveli has 812

195. According to the census 2001, which one of the following Indian States has the maximum population in India after Uttar Pradesh?

Explanation: B. According to 2001 census, Maharashtra had the maximum population in India after Uttar Pradesh. The total population in Maharashtra is 193,977,000.

196. Which one of the following statements is not correct?

Explanation: A. Western Ghats are are relationship higher in the Southern region.

197. Gandhi Sagar Dam is a part of which one of the following?

Explanation: A. Gandhi Sagar Dam is situated on the river Chambal in 1960, near Bhanpura of Madhya Pradesh.

198. Which one of the following is the correct sequence of hills starting from the north and going towards the south?

Explanation: C. The correct sequence of the given hills starting from north and going towards to south is Nallamalai Hills, Javadi Hills, Nilgiri Hills, Anamalai Hills

199. Which one of the following is not a Biosphere reserve?

Explanation: B. Nallamalai is not a biosphere reserve. It is hill of Eastern ghats which stretches over Kurnool, Mahabubnagar, Guntur and Kadapa districts of the state of Andhra Pradesh.

200. Which one of the following is the correct sequence of Indian cities in the decreasing order, of their normal annual rainfall?

Explanation: C. The correct sequence of the given Indian cities in the decreasing order of their normal annual rainfall is Kochi (3228.3 mm)

201. Which of the following states border Uttar Pradesh?
  1. Punjab
  2. Rajasthan
  3. Chhattisgarh
  4. Jharkhand

Select the correct answer using the codes given below:

Explanation: B. Uttar Pradesh is bounded by Haryana, Delhi, Rajasthan in west; Madhya Pradesh in the south and Bihar in the east. The northern boundary is with Nepal.

202. Match items in the List-I with List-II and select the correct answer using the codes given below the lists.
  1. A. Kothagudem 1. Andhra Pradesh
  2. B. Raichur 2. Gujarat
  3. C. Mettur 3. Karnataka
  4. D. Wanakbori 4. Tamil Nadu

Codes:

Explanation: B. A-1; B-3; C-4; D-2.

203. Consider the following statements:
  1. Silent Valley National Park in the Nallamalai range.
  2. Pathrakkadavu Hydroelectric project is proposed to be built near the Silent Valley National Park.
  3. The Kunthi river originates in Silent Valley's rainforests.

Which of the statements given above is/are correct?

Explanation: C. Silent Valley National Park is situated in Nilgiri Hills of Western Ghats. The park is bound by Attappadi reserved forest to the east and vested forest of Palaghat division and Nilamber division to the south-west respectively.

204. Match List-I with List-II and select the correct answer using the codes given below the lists:

Explanation: B. A-4; B-3; C-2; D-1.

205. Consider the following sites /monuments:
  1. Champaner-Pavagadh Archaeological Park
  2. Chhatrapati Shivaji Railway Station, Mumbai
  3. Mahallapuram
  4. Sun Temple (Konark Temple)

Which of the above are included in the World Heritage List of UNESCO?

Explanation: D. All the monuments / sites are included in World Heritage list of UNESCO. Sun Temple of Orissa, group of monuments at Mahabalipuram were included in 1985. Champaner

206. In which one of the following oceans Diamantine Trench is situated?

Explanation: C. Diamantine Trench is istuated in south-east part of Indian Ocean. The depth of Diamantine Trench is 8047 m.

207. Assertion (A): To orbit around the Sun the planet Mars takes lesser time than the time taken by the earth. Reason (R): The diameter of the planet Mars is less than that of earth.

Codes:

Explanation: D. A is incorrect as the mars revolution round the sun takes 1.88 years of Earth's revolution. The diametre of earth is almost double to the diameter of mars.

208. Bermuda Triangle extends up to which of the following places?
  1. Southern Florida
  2. Puerto Rico
  3. Hawaiian Islands

Select the correct answer using the codes given below:

Explanation: B. Bermuda Triangle is a strange and mysterious location in southern Atlantic Ocean. It is roughly in the shape of a triangle which extends upto South Florida, Pruto Rico and Bermuda Island.

209. Claims to the historical Macedonian territory have been a bone of contention between which of the following countries?

Explanation: B. Bulgaria & Greece are in dispute over the Macedonian territory.

210. Huangpu River flows through which one of the following cities?

Explanation: C. The Huangpu river is the largest river in Shanghai in China. It is 113.07 km long 400 metres wide and 9 metres deep. Shanghai gets most of its drinking water from Huangpu.

211. Which one of the following countries is the leading producer of uranium?

Explanation: B. As per a 2016 report, Kazakhstan is the largest producer of uranium with a total output of 24,575 tonnes, accounting for 39 percent of global uranium supply in 2016. But among the given alternatives in the question, canada is the largest producer of uranium in world. Largest uranium producers :(2016) 1. Kazakhstan; 2. Canada; 3. Australia; 4. Niger; 5. Namibia; 6. Russia.

212. Consider the following statements:
  1. Length of a terrestrial mile is lesser than that of a nautical mile.
  2. Harmattan is a dusty land-wind of the East African Coast.
  3. Greece and Albania form a part of the Iberian Peninsula.

Which of the statement(s) given above is/are correct?

Explanation: D. One terrestrial mile (1609 metres) is lesseer than one nautical mile (1852 meters). The Harmattan is a season in the west African subcontinent, which occurs the end of November and the middle of March. It is characterized by the dry and dusty north easterly trade wind which blows from the Sahara Desert over west Africa into the Gulf of Guinea. The Iberian peninsula includes Portugal, Spain, Andorra and Gibraltar.

213. Through which one of the following groups of countries does the Equator pass?

Explanation: B. Colombia, Kenya and Indoneisa are countries through which the equator passes.

214. Which one of the following pairs is not correctly matched?

Explanation: A. Baikonur is world's first and largest operational space launch facility. It is located in Kazakhstan and not in Russia.

215. Other than India and China, which one of the following groups of countries border Myanmar?

Explanation: D. Thailand, Laos and Bangladesh are countries bordering Myanmar.

216. Consider the following statements concerning the Indian Railways.
  1. The Head Quarters of the North Western Railway are located at Jodhpur.
  2. "Indrail Pass"
  3. Fairy Queen is a train using the world's oldest working engine and the Indian Railways conducts a journey of wildlife and heritage sites on it.

Which of the statements given above is/are correct?

Explanation: B. Correct option is B.

217. Consider the following statements:
  1. Sikkim has the minimum area among the 28 Indian States (Delhi and Pondicherry not included).
  2. Chandigarh has the highest literacy rate among Pondicherry, National Capital Territory of Delhi and other Union Territories.
  3. Maharashtra has the highest population after Uttar Pradesh among the 28 Indian States (Delhi and Pondicherry not included).

Which of the statement(s) given above is/are correct?

Explanation: D. Maharashtra is ranked 2nd after Uttar Pradesh among 28 Indian states. So, option '3' is correct. But option '1' and '2' are incorrect, because Goa is the smallest state based on area (i.e., 3702 sq km), not the Sikkim. Highest literacy rate is of Union territory is Lakshadweep i.e. 86.7%.

218. Match List-I (Centre of Handicrafts) with List-II (State) and select the correct answer using the codes given below the lists:
  1. A. Mon 1. Arunachal Pradesh
  2. B. Nalbari 2. Assam
  3. C. Naisghat 3. Meghalaya
  4. D. Tura 4. Nagaland

Codes :

Explanation: A. A-4; B-2; C-1; D-3.

219. Consider the following statements:
  1. Petronet LNG Ltd. is setting up another LNG terminal at Mangalore.
  2. The Head Office of the Dredging Corporation of India is at Vishakhapatnam.
  3. The Narwapahar Mine is operated by the Uranium Corporation of India Limited.

Which of the statements given above are correct?

Explanation: C. Correct option is C.

220. Consider the following statements:
  1. Appellate Tribunal for electricity has been established by each state government in India.
  2. One of the component of the Accelerated Power Development and Reforms Programme (APDRP) is up gradation of sub-transmission and distribution system for electricity in India.

Which of the statement(s) given above is/are correct?

Explanation: B. By virtue of section 110 of the Electricity Act 2003, an Appellate Tribunal for Electricity having jurisdiction throughout India has been set up to hear appeals or original petitions against the order of the Adjudicating officer. But in states like Delhi and Orissa electricity has been privatized. So, the statement '1' is wrong. It is not present in Jammu and Kashmir.

221. Match List-I (Valley) with List-II (State) and select the correct answer using the codes given below the lists:
  1. A. Markha Valley 1. Sikkim
  2. B. Dzukou Valley 2. Himachal Pradesh
  3. C. Sangla Valley 3. Jammu & Kashmir
  4. D. Yumthang Valley 4. Nagaland

Codes:

Explanation: D. A-3; B-4; C-2; D-1.

222. Which one of the following statements is not correct?

Explanation: C. Cauvery is a river of southern India, rises on Brahmagiri Hill in Western Ghats in Coorg district of Karnataka.

223. Match List-I (National Park/Wildlife Sanctuary) with List-I (Nearby Town) and select the correct answer using the codes given below the lists:

Explanation: C. A-4; B-2; C-5; D-1.

224. From North towards South, which one of the following is the correct sequence of the given rivers in India?

Explanation: B. The correct sequence of the rivers in India from north towards south is Shyok

225. Assertion (A): The percentage of net sown area in the total area of Andhra Pradesh is less as compared to that of West Bengal. Reason (R): The soil of most of Andhra Pradesh is laterite.

Explanation: C. Laterite soil is mostly found in areas of high rainfall and temperature with alternate dry and wet Periods it is suitable for growing rice, ragi and sugarcane.

226. What is the average distance (approximate) between the sun and the earth?

Explanation: D. The actual average distance between the sun and the earth is 152

227. Consider the following statements:
  1. Either of the two belts over the oceans at about 30
  2. Horse latitudes are low pressure belts.

Which of the statements given above is/are correct?

Explanation: A. The horse latitudes are located between latitude 30

228. Consider the following statements:
  1. Annual range of temperature is greater in the Pacific Ocean than that in the Atlantic Ocean.
  2. Annual range of temperature is greater in the Northern Hemisphere than that in the Southern Hemisphere.

Which of the statements given above is/are correct?

Explanation: B. The maximum and minimum annual temperatures of Ocean water are recorded in August and February respectively in the northern hemisphere. The annual range of temperature is higher in the enclosed seas than in the open sea. The Atlantic Ocean records relatively higher annual range of temperature than the Pacific Ocean. Since the Northern hemisphere has more landmass as compared to the Southern hemisphere, the annual range of temperature will be greater in northern hemisphere.

229. Where is Copacabana Beach located?

Explanation: C. Copacabana Beach is located in Rio de Janeiro in Brazil.

230. Which one of the following cities does not have the same clock time as that of the other three cities at any given instant?

Explanation: D. London, Lisbon and Accra in Ghana are close to Greenwich Meridian. But, Addis Ababa is Situated on the eastern part of Africa (i.e., longitude 38.74 E). So, Addis Ababa has a different clock time than that of London, Lisbon and Accra.

231. Match List I with List II and select the correct answer using the code given below the lists:
  1. A. Bangkok 1. Irrawaddy
  2. B. Phnom-Penh 2. Mekong
  3. C. Hanoi 3. Menam (Chao Phraya)
  4. D. Yangon 4. Red River

Code:

Explanation: A. A-3; B-2; C-4; D-1.

232. Which one among the following rivers is the longest?

Explanation: A. Amazon is the longest river in the world. It originates from Andes mountain in Peru.

233. Through which one of the following Straits, does a tunnel connect the United Kingdom and France?

Explanation: C. Strait of Dover connects United Kingdom and France. It is about 32 km in length.

234. The largest coral reef in the world is found near the coast of which one of the following countries?

Explanation: A. The largest coral reef is the Great Barrier Reef, located just off the north-eastern coast of Australia. The 1200 mile (1900 km) long reef is protected as a Marine Park.

235. In which one of the following districts, have large reserves of diamond-bearing Kimberlite been discovered in the recent past?

Explanation: B. Payali and Behradein in Raipur, Chhattisgarh has large reserves of diamond bearing kimberlite. It has 28% of the total diamond reserve of India.

236. With reference to the steel industry in India in the recent times, consider the following statements:
  1. Vizag Steel Plant (RINL) has been declared a Mini Ratna.
  2. Merger of IISCO with SAIL has been completed.

Which of the statements given above is/are correct?

Explanation: C. Vizag Steel Plant was declared as a Mini Ratna company on 11th Feb 2006. Merger of IISCO with SAIL has been completed on 29th September 2004.

237. The Stilwell Road, built in 1940s, which was recently in news, connects which of the following?

Explanation: B. The Stilwell Road built in 1940s by Americans to connect Ledo in India and Kunming in China via Myanmar during Second World War.

238. Which one of the following is also known as Top Slip?

Explanation: D. Indira Gandhi Wildlife Sanctuary and National Park is known as top slip. It is a part of Western Ghats is located above 300 metres from the sea level on the Anamalai mountain ranges.

239. Where are Shevaroy hills located?

Explanation: D. Shevaroy hill is situated near Salem of Tamil Nadu. This hill range covers an area of fifty square kilometers.

240. Assertion (A): River Kalinadi is an east-flowing river in the southern part of India. Reason (R): The Deccan Plateau is higher along its western edge and gently slopes towards the Bay of Bengal in the east.

Explanation: D. River Kalinadi is a west flowing river in the southern part of India. So, the assertion is wrong. Deccan Plateau has elevation ranging from 1,500 to 2,500 ft and gently slopes towards the Bay of Bengal in the east. So, the reason is correct.

241. Which one of the following National Highways passes through Maharashtra, Chhattisgarh and Orissa?

Explanation: C. National Highways passes through Gujarat, Maharashtra, Chhatisgarh, Orissa, Jharkhand and West Bengal. This road is 1,949 km long.

242. Consider the following statements:
  1. Balaghat is known for its diamond mines.
  2. Majhgawan is known for its manganese deposits.

Which of the statements given above is/are correct?

Explanation: D. Balaghat of Madhya Pradesh is known for its manganese mines. Majhgawan is famous for diamond.

243. Which one of the following rivers originates in Amarkantak?

Explanation: C. The Narmada river originates from a tank 1057 m high west of Amarkantak plateau in Madhya Pradesh. River Damodar originates from Chhota Nagpur plateau, Mahanadi originates from Bastar plateau and Tapti originates from Satpura hills.

244. Which one among the following major Indian cities is most eastward located?

Explanation: C. Lucknow is the city most east in the list.

245. Out of the four southern States: Andhra Pradesh, Karnataka, Kerala and Tamil Nadu, which shares boundaries with the maximum number of Indian States?

Explanation: C. Both Andhra Pradesh and Karnataka shares boundaries with maximum number of Indian states.

246. Match List I with List II and select the correct answer using the code given below the lists:
  1. A. BALCO 1. Hirakud
  2. B. HINDALCO 2. Korba
  3. C. Indian Aluminium Company 3. Koraput
  4. D. NALCO 4. Renukoot

Code:

Explanation: B. A-2; B-4; C-1; D-3

247. Which one of the following is located in the Bastar region?

Explanation: D. Indravati National Park is located in the Bastar region. The park is situated at the distance of 97.4 km Bastar.

248. In which State is the Guru Shikhar Peak located?

Explanation: A. Guru Shikhar Peak is the highest point in Rajasthan. The altitude of peak 5676 feet (1722 mt).

249. Match List I with List II and select the correct answer using the code given below the lists:
  1. A. Betul 1. Indravati
  2. B. Jagdalpur 2. Narmada
  3. C. Jabalpur 3. Shipra
  4. D. Ujjain 4. Tapti

Code:

Explanation: B. A-4; B-1; C-2; D-3.

250. Consider the following statements:
  1. In India, Red Panda is naturally found in the Western Himalayas only.
  2. In India, Slow Loris lives in the dense forests of the North East.

Which of the statements given above is/are correct?

Explanation: B. Red Panda is found in mountains of Nepal, northeastern India, China, Bhutan and Slow Loris lives in dense forest of north-east, dense forest of Assam. Red Panda is found in Eastern Himalayas, China, Bhuan and slow loris in the dense forest of north-east Assam.

251. Which one among the following States of India has the lowest density of population?

Explanation: C. Population density of Arunachal Pradesh is 13, Himachal Pradesh is 109, Meghalaya is 103, and Sikkim is 76.

252. Which one of the following is the correct sequence in the decreasing order of production (in million tones) of the given foodgrains in India?

Explanation: D. Production of rice is 96.43 MT, wheat is 78.4 MT, pulses is 15.11 MT and coarse cereals are 40.73 MT in India. So, the correct sequence in decreasing order is Rice

253. Assertion (A): There are no tea plantations in any African country. Reason (R): Tea plants need fertile soil with high humus.

Explanation: D. In the 20th century tea was being grown in 3 African countries like Tanzania, Kenya and Uganda. So, the assertion is wrong. But the reason is correct because tea plants need highly organic or loamy soils.

254. Dalbergia species is associated with which one of the following?

Explanation: D. Rosewood is associated with Dalbergia species. Dalbergia species are used as food plants and fragnant wood in aromatic oils.

255. Which one of the following places was associated with Acharya Vinoba Bhave

Explanation: C. Bhoodan Movement was a voluntary land reform movement in India started by Acharya Vinoba Bhave on April 18, 1951 started at Pochampalli village, in Telangana (Andhra Pardesh).

256. In order of their distances from the Sun, which of the following planets lie between Mars and Uranus?

Explanation: B. Jupiter and Saturn are located between Mars and Uranus.

257. In which one of the following is Malta located?

Explanation: B. Malta is in the centre of Mediterranean sea. Italy is in south and Libya is located north of Malta. Its area is 246 sq km.

258. Which of the following cities is nearest to the equator?

Explanation: D. Singapore is 200 km north of equator.

259. Which one of the following straits is nearest to the International Date Line?

Explanation: B. Bering Strait is nearest to the International Date line. Located 1.5 km away from international date line.

260. Which of the following countries share borders with Moldova?
  1. Ukraine
  2. Romania
  3. Belarus

Select the correct answer using the code given below:

Explanation: A. Maldova is a land locked country in Eastern Europe. It shares boundaries with Romania in the west and Ukraine in the north, east and south.

261. Conisder the following statements:
  1. Ajman is one of the seven emirates of the UAE.
  2. Ras al-Khaimah was the last Sheikhdom to join the UAE.

Which of the statements given above is/are correct?

Explanation: C. Ajman is one of the seven emirates constituting the United Arab Emirates with an area of 260 sq kilometer. Ras al-Khaimah was the 7th and last Sheikhdom to join UAE in 1972.

262. Which two countries follow China and India in the decreasing order of their population?

Explanation: B. USA and Indonesia.

263. As per India's National Population Policy, 2000, by which one of the following years is it our long-term objective to achieve population stabilization?

Explanation: C. As per India's National Population Policy - 2000, by 2045 India will achieve population stability which means that the size of population will not go up.

264. What is the approximate percentage of persons above 65 years of age in India

Explanation: C. The correct answer is (C).

265. Amongst the following States, which one has the highest percentage of rural population to its total population (on the basis of the Census, 2001)?

Explanation: A. On the basis of 2001 census, 9.8% of population Himachal Pradesh, 10.5% of population of Bihar, 15% of population of Orissa & 20.8 % of population of Uttar Pradesh live in rural areas.

266. Among the following, which one has the minimum population on the basis of data of Census of India, 2001?

Explanation: D. Population of Sikkim is 54,0851. Population of Chandigarh is 901,000, Population of Pondicherry is 974000 and population of Mizoram is 889,000.

267. Which one of the following is not essentially a species of the Himalayan vegetation?

Explanation: B. Mahogany is a species of tree found throughout the Caribbean, central and south America but not in Himalayas.

268. Out of all the biosphere reserves in India, four have been recognized on the World Network by UNESCO. Which one of the following is not one of them?

Explanation: B. Kanchenjunga.

269. With which one of the following rivers is the Omkareshwar Project associated?

Explanation: B. Omkareshwar Project is associated with Narmada river.

270. In India, how many states share the coastline?

Explanation: C. 9 states of India have a coastline. Gujrat, Maharastra, Karnataka, Kerla, Tamil Nadu, Andhra Pradesh, Odisha, West Bengal.

271. Which of the following hills are found where the Eastern Ghats and the Western Ghats meet?

Explanation: C. Nilgiri hills are at the junction of the eastern and western ghats of the Sahayadri hills. The heights of the hills range varies between 2,280 and 2,290 metres.

272. Consider the following pairs:
  1. Chambal Narmada
  2. Son Yamuna
  3. Manas Brahmaputra

Which of the pairs given above is/are correctly matched?

Explanation: D. Tributaries of Brahmaputra in India the Manas, Pagladiya, Puthimari, Dhanisri, Jia Bhariti and Subansiri. Manas is a tributary of Brahmaputra Chambal is the chief tributary of Yammuna and sone is a tributary of Ganga.

273. Consider the following statements:
  1. Salt-water crocodile is found in the Andaman and Nicobar Islands.
  2. Shrew and Tapir are found in the Western Ghats of the Malabar region.

Which of the statements given above is/are correct?

Explanation: C. Salt water crocodile was common and found throughout the Andaman and Nicobar Islands living in the mangrove habitats. Shrew and Tapir are found in western ghats of the Malabar region.

274. In which one of the following states are Namchik Namphuk coalfields located?

Explanation: A. Namchik-Namphuk coalfields are located in Arunachal Pradesh

275. Which of the following minerals found in a natural way in the state of Chhattisgarh?
  1. Bauxite
  2. Dolomite
  3. Iron ore
  4. Tin

Select the correct answer using the code given below:

Explanation: D. The minerals deposits in Chhatisgarh is Bauxite (96 million ton), Dolomite (606 million ton) and iron ore (2336 million ton). Tin is not found in Chhatisgarh.

276. On which one of the following rivers is the Tehri hydropower complex located?

Explanation: B. The Tehri Hydropower complex is located on the Bhagirathi river of Uttar Pradesh.

277. Which of the following pairs are correctly matched?
  1. Kapildhara Falls : Godavari
  2. Jog Falls : Sharavathi
  3. Shivasamudram Falls : Cauvery

Select the correct answer using the code given below:

Explanation: B. Shivasamudram falls is located on river Cauvery. Jog waterfall is the highest waterfall in India of 253 metres on Sharavathi river.

278. Which of the following pairs are correctly matched?
  1. Damanganga : Gujarat
  2. Girna : Maharashtra
  3. Pamba : Kerala

Select the correct answer using the code given below:

Explanation: D. Damanganga water project is in Gujarat. Girna irrigation project is in Jalgaon area of Maharashtra. Pamba river is located in Kerala.

279. Where are Tapovan and Vishnugarh hydroelectric project located?

Explanation: C. Tapovan and Vishnugarh hydroelectric project are located in Chamoli district of Uttarakhand.

280. Which of the following are among the million-plus cities in India on the basis of data of the Census, 2001?
  1. Ludhiana
  2. Kochi
  3. Surat
  4. Nagpur

Select the correct answer using the code given below:

Explanation: D. According to the Census of 2001 of India, the population of Ludhiana is 1,398,467, Surat is 2,433,835 and Nagpur is 2,052,066.

281. Which one among the following has the maximum number of National Parks?

Explanation: A. Andaman and Nicobar has 9 national parks; Assam has 6, Arunachal Pradesh and Maghalaya both have 2 each.

282. Which one amongst the following has the largest livestock population in the world?

Explanation: C. India has largest livestock population in the world, having 57% of world

283. Consider the following statements:
  1. Chikmangalur is well-known for sugar production.
  2. Mandya is well-known as a coffee producing region.

Which of the statements given above is/are correct?

Explanation: D. Chikmangalur is famous for coffee. Mandya is famous for sugar and paper production.

284. Which one of the following planets has largest number of natural satellites or moons?

Explanation: A. Jupiter has maximum moons accounting to 63 whereas Satun has 61, Mars has 2 and Venus has none.

285. In the structure of planet Earth, below the mantle, the core is mainly made up of which one of the following?

Explanation: C. The core of earth is made up of nickle and iron.

286. Consider the following countries:
  1. Australia
  2. Namibia
  3. Brazil
  4. Chile

Through which of the above does the Tropic of Capricorn pass?

Explanation: D. The Tropic of Capricorn runs through 10 countries: Namibia, Botswana, South Africa, Mozambique, Madagascar, Australia, Chile, Argentina, Paraguay and Brazil.

287. Which one of the following pairs is not correctly matched?

Explanation: A. Located in north eastern Germany on the banks of the rivers spree and Havel, Berlin is the centre of the Berlin - Brandenburg Metropolitan Region.

288. Which one among the following South Asian countries has the highest population density?

Explanation: A. Overall population density of India is 324. Nepal is 102, Pakistan is 146, Sri Lanka is 280 person per square kilometers.

289. Match List I with List II and select the correct answer using the code given below the lists:
  1. A. Great Victoria Desert 1. Australia
  2. B. Grand Canyon 2. Canada
  3. C. Lake Winnipeg 3. New Zealand
  4. D. Southern Alps 4. USA

Code:

Explanation: B. A-1; B-4; C-2; D-3.

290. Cape Canaveral, the site from which space shuttles are launched is located on the coast of:

Explanation: A. Cape Canaveral is the part of Florida's coast.

291. Which among the following has the world's largest reserves of Uranium?

Explanation: A. Australia has the world's largest uranium reserves. Approximately 24% of the planet's uranium is present in Australia.

292. Which one of the following is the country's administrative capital new federal administrative centre of Malaysia?

Explanation: C. Putrajaya is new federal administrative centre of Malaysia.

293. Consider the following regions:
  1. Eastern Himalayas
  2. Eastern Mediterranean region
  3. North-Western Australia

Which of the above is/are Biodiversity Hotspot(s)?

Explanation: B. Eastern Himalayas and Eastern Mediterranean region are Biodiversity hotspots.

294. In India, which one of the following states has the largest inland saline wetland?

Explanation: D. Rajasthan has the largest inland saline wetland, area of Sambhan Salt lake.

295. In India, the ports are categorized as major and nonmajor ports. Which one of the following is a non-major port?

Explanation: B. Dahej is a not a major port of India.

296. In which one of the following places is the Shompen tribe found?

Explanation: B. The Shompen tribe is one of two Mongloid tribes found in Nicobar Island.

297. The Dul Hasti Power Station is based on which one of the following rivers?

Explanation: B. The Dul Hasti Power Station is on the Chenab river in Jammu and Kashmir.

298. Consider the following statements:
  1. In the world, the tropical deserts occur along the western margins of continents within the trade wind belt.
  2. In India, the east Himalayan region gets high rainfall from north-east winds.

Which of the statements given above is/are correct?

Explanation: A. In India, the south-west monsoon causes high rainfall in east Himalayan region.

299. Consider the following statements:
  1. Infant mortality rate takes into account the death of infants within a month after birth.
  2. Infant mortality rate is the number of infant deaths in a particular year per 100 live births during that year.

Which of the above statements is/are correct?

Explanation: D. Infant mortality rate indicates the number of infant death under one years of age per 1000 live birth under one year of age.

300. Consider the following statements:
  1. Between Census 1951 and Census 2001, the density of the population of India has increased more than three times.
  2. Between Census 1951 and Census 2001, the annual growth rate (exponential) of the population of India has doubled.

Which of the statements given above is/are correct?

Explanation: D. The rate of population density increase in India is as follows: Density 1951

301. Which one of the following rivers does not originate in India?

Explanation: D. Sutlej originates from the central Sulaiman range in Pakistan.

302. At which one of the following places do two important rivers of India originate; while one of them flows towards north and merges with another important rivers flowing towards Bay of Bengal, the other one flows towards Arabian Sea?

Explanation: A. Amarkantak is the origin of River Narmada and Sone.

303. Consider the following statements:
  1. India does not have any deposits of Thorium.
  2. Kerala's monazite sands contains Uranium.

Which of the statements given above is/are correct?

Explanation: D. India has 12% of thorium deposits of the world.

304. Consider the following statements:
  1. There are no east flowing rivers in Kerala.
  2. There are no west flowing rivers in Madhya Pradesh.

Which of the statements given above is/are correct?

Explanation: D. Three east flowing rivers found in Kerala is Kavari, Tapti, Narmada and Mahi rivers flows westward, and also flows through Madhya Pradesh. East flowing rivers of Kerala are Kabani, Bhavani and Pambar. The west flowing rivers in M.P. are Narmada, Tapti, Mahi.

305. Sun's halo is produced by the refraction of light in:

Explanation: C. Sun halo also known as 22 degree halo, the ring is caused by sunlight passing through ice crystals in cirrus clouds with in the Earth's atmosphere.

306. Consider the following statements:
  1. Ocean currents are slow-surface movement of water in the ocean.
  2. Ocean currents assist in maintaining the Earth's heat balance
  3. Ocean currents are set in motion primarily by prevailing winds
  4. Ocean currents are affected by the configuration of the ocean

Which of these statements are correct?

Explanation: B. Ocean currents are not slow most of the time and can be subsurface current also. So, statement '1' is wrong. But '2', '3' and '4' statements are correct.

307. Consider the following statements:
  1. In equatorial regions, the year is divided into four main seasons
  2. In Mediterranean region, summer months receives more rain.
  3. In China type climate; rainfall occurs throughout the year
  4. Tropical highlands exhibit vertical zonation of different climates

Which of these statements are correct?

Explanation: D. 1st statement is wrong: Equational region has only two seasons. 2nd statement : Mediterranean gets rainfall during winter season is also wrong.

308. Consider the following ecosystems:
  1. Taiga
  2. Tropical Evergreen
  3. Tropical Deciduous
  4. Tundra

The correct sequence in decreasing order of the Albedo values of these ecosystems is:

Explanation: B. According to ecosystem, the maximum albedo would be of Tundra, then Taiga, then Tropical Deciduous then Tropical ever green.

309. Assertion (A): 60

Explanation: C. A is true but R is false. Sub-polar low pressure is located between 60

310. Assertion (A): The surface winds spiral inwards upon the centre of the cyclone. Reason (R: Air descends in the centre of the cyclone.

Explanation: A. Air begins to slowly descend in the centre of the storm, creating a rain-free area. This is a newly formed eye. On land, the centre of the eye is, by for, the calmest part of the storm.

311. Consider the following climatic and geographical phenomena:
  1. Condensation
  2. High temperature and humidity
  3. Orography
  4. Vertical wind

Thunder cloud development is due to which of these phenomena?

Explanation: D. The thunder clouds develop by the above climatic and geographic phenomena. High temperature and humidity causes the wind to rise vertically up and due to orography and pressure of mountains these winds get condensed and form cumulonimbus clouds or thunder clouds.

312. For short-term climatic predictions, which one of the following events, detected in the last decade, is associated with occasional weak monsoon rains in the Indian sub-continent?

Explanation: C. E1 Nino and La Nina are opposite phases of what is known as the E1 Nino-southern Oscillation (ENSO) cycle. The ENSO cycle is a scientific term that describes the fluctuations in temperature between the ocean and atmosphere in the east-central. Impact of E1 -Nino: Normal or High rainfall in Eastern/Central Pacific and Drought or Scant rainfall in western Pacific/Asia.

313. Which one of the following is not a lagoon?

Explanation: C. Periyar Lake is favoured by the construction of the dam across the Mulloperiyar River. It is an artificial lake in the middle of the periyar wildlife sanctuary.

314. Which one of the following statements is correct with reference to our solar system?

Explanation: A. The earth is the densest of all the planet. Density of Earth is 5.52, Venus

315. Consider the following countries:
  1. Brazil
  2. Indonesia
  3. Japan
  4. Russia

What is the descending order of the size of the following countries population-wise?

Explanation: C. Indonesia>Brazil>Russia>Japan.

316. Consider the following countries of South Asia:
  1. Bangladesh
  2. India
  3. Pakistan
  4. Sri Lanka

The descending order of literacy status of these countries is:

Explanation: C. According to UNESCO; 2015, the literacy rate of given countries are - Paskistan - 56.4%; India - 72.1%; Bangladesh - 72.8%; Srilanka - 92.6%.

317. Consider the following countries:
  1. Albania
  2. Bosnia Herzegovina
  3. Croatia
  4. Macedonia

Which of these countries has/have Adriatic Sea as a boundary?

Explanation: B. Bosnia, Hercegovina, Alabania, Croatia have Adriatic sea as a boundary. Macedonia is far away from Adriatic sea.

318. The correct sequence of the eastward flowing rivers of the peninsular India from north to south is:

Explanation: A. The correct sequence of eastward flowing river of the peninsular India from north to south is Subarnarekha, Mahanadi, Godavari, Krishna, Pennar, Cauvery and Vagai.

319. India's population growth during the 20th century can be classified into four distinct phases. Match List-I (Period) with List-II (Phase) and select the correct answer using the codes given below the lists:
  1. A. 1901-1921 1. Steady growth
  2. B. 1921 -1951 2. Rapid high growth
  3. C. 1951-1981 3. Stagnant growth
  4. D. 1981 -2001 4. High growth with definite signs of slowdown

Codes:

Explanation: C. 1901

320. With reference to Indian transport systems, consider the following statements:
  1. Indian railway system is the largest in the world
  2. National Highways cater to 45 percent of the total and transport demand
  3. Among the states, Kerala has the highest density of surface road
  4. National Highway No. 7 is the longest in the country

Which of these statements are correct?

Explanation: D. Indian railway is the fourth largest railway in the world after USA, Russia and China. National Highways is nearly 2% (1.96%) of total roads, but it carries nearly 40

321. The average annual temperature of a meteorological station is 26

Explanation: B. The climatic condition of Chennai is tropical savanna type where the average annual temperature of meterological station is 26

322. The sea coast of which one of the following states has become famous as a nesting place for the giant Olive Ridley turtles from South America?

Explanation: C. The sea coast of Orissa is famous as a nesting place for giant olive Ridley turtles from South America.

323. With reference to India, which one of the following statements is not correct?

Explanation: B. As per data of 2010

324. Open stunted forests with bushes and small trees having long roots and sharp thorns or sharp thorns or spines are commonly found in:

Explanation: D. Open stunted forests with bushes and small trees having long roots and sharp thorns or spines are commonly found in the area where the rainfall is less than 80 cm. Such areas are found in Rajasthan, Gujarat, Haryana and western Andhra Pradesh.

325. Match List-I (Mangrove) with List-II (State) and select the correct answer using the codes given below the lists:

Explanation: D. Achra Ratnagiri Mangrove is in Maharashtra, Coondapur Mangrove is in Karnataka, Pichavaram Mangrove is in Tamil Nadu and Vembanad Mangrove is in Kerala.

326. With reference to Indian agriculture, which one of the following statements is correct?

Explanation: A. In Indian agriculture about 90% of the area under pulse is rainfed, but pulses need less water for cultivation so, the production of pulse hampers.

327. Consider the following high yielding varieties of crops in India:
  1. Arjun
  2. Jaya
  3. Padma
  4. Sonalika

Which of these are of wheat?

Explanation: C. Arjun and Sonalika are hybrid variety of wheat produced by Indian Council of Agricultural Research.

328. Consider the following statements:
  1. On the planet Earth, the fresh water available for use amounts to about less than 1% of the total water found.
  2. Of the total fresh water found on the planet Earth 95% is bound up in polar ice caps and glaciers.

Which of the statements given above is/are correct?

Explanation: B. About 2.75 percent of water is fresh water an earth and about 95% of it is frozen in glaciers.

329. A new type of El Nino called El Nino Modoki appeared in the news. In this context, consider the following statements:
  1. Normal El Nino forms in the Central Pacific Ocean whereas El Nino Modoki forms in Eastern Pacific Ocean
  2. Normal El Nino results in diminished hurricanes in the Atlantic Ocean but El Nino Modoki results in a greater number of hurricanes with greater frequency.

Which of the statements given above is/are correct?

Explanation: B. Normal El Nino forms in south

330. Which one of the following reflects back more sunlight as compared to other three?

Explanation: C. The Albedo of Snow is hightest.

331. A geographic region has the following distinct characteristics:
  1. Warm and dry climate
  2. Mild and wet winter
  3. Evergreen Oak trees

The above features are distinct characteristics of which one of the following regions?

Explanation: A. Mediterranean climate have mild, rainy winter and hot, dry summers and Evergreen, Oak trees.

332. What causes wind to deflect toward left in the Southern Hemisphere?

Explanation: C. Rotation of the earth causes wind to deflect towards left in the Southern Hemisphere.

333. Which one of the following can come across if one travels through the Strait of Malacca?

Explanation: D. Malacca strait is between the Indonesian island of Sumatra and the Malaya Peninsula. Singapore lies on the tip of the Malay Peninsular.

334. Consider the following statements:
  1. The boundaries of a National Park are defined by legislation.
  2. A Biosphere Reserve is declared to conserve a few specific species of flora and fauna.
  3. In a Wildlife Sanctuary, limited biotic interference is permitted.

Which of the statements given above is/are correct?

Explanation: C. The fix boundary of a National Park is described in the Wildlife Protection Act, 1972. The legislation states the actual area of the National Park which is notified by the state government. A biosphere reserve conserves an ecosystem and not just few specific species of plants and animals.

335. Following are the characteristics of an area in India:
  1. Hot and humid climate
  2. Annual rainfall 200 cm
  3. Hill slopes up to an altitude of 1100 metres
  4. Annual range of temperature 15

Which one among the following crops are you most likely to find in the area described above?

Explanation: C. C is correct option.

336. With reference to the mineral resources of India, consider the following pairs:
  1. Copper
  2. Nickel
  3. Tungsten

Which of the pairs given above is/are correctly matched?

Explanation: B. B is correct option.

337. Consider the following pairs:
  1. Bhiterkanika, Odisha
  2. Desert National Park
  3. Eravikulam, Kerala

Which of the pairs given above is/are correctly matched?

Explanation: B. Hoolak found in North eastern reigon of India. It covers 38% of the forest area. Hoolak Gibbon is found in North Eastern region of India.

338. In India, which type of forest among the following occupies the largest area?

Explanation: C. In India tropical moist deciduous forest occupies the largest area. It includes Deccan plateau, the north eastern region of Deccan plateau and the lower slopes of Himalayas and covers the part of Andhra Pradesh, Tamil Nadu, Orissa, Bihar, Chhatisgarh, Jharkhand, Karnataka and Kerala. In India, tropical moist deciduous forest covers 38% of the total forest area. It covers the region of Bihar, Uttar Pradesh, Odisha, Andhra Pradesh, Tamil Nadu, Jharkhand, Karnataka and Kerala.

339. With reference to, the river Luni, which one of the following statements is correct?

Explanation: D. Luni river originates from Aravalli range and flows in south west direction through the hills and finally ends up in the marshy land of Rann of Kutch.

340. Rivers that pass through Himachal Pradesh are:

Explanation: D. Rivers like Beas, Chenab, Ravi, Sutlej and Yamuna passes through Himachal Pradesh.

341. When you travel in certain parts of India, you will notice red soil. What is the main reason for this colour?

Explanation: C. Red soil in India is largely found in Deccan plateau. Red soil is less clayey and sandier in nature and has a rich content of iron and small amount of humus. Red soil is also known as yellow soil. The redness is the soil is due to Jerric oxide, is 2nd largest soil found in India from south of Bundelkhand to North of Raj mahal hill.

342. Which one of the following is the appropriate reason to considering the Gondwana rocks as most important of rock systems of India?

Explanation: B. More than 90% of India's coal reserves are found in Gondwana rock system.

343. Which one of the following pairs is not correctly matched?

Explanation: B. Kolleru lake is one of the largest shallow fresh water lakes in Asia, is located between Krishna delta and the Godavari in Andhra Pradesh.

344. If there were no Himalayan ranges, what would have been the most likely geographical impact on India?
  1. Much of the country would experience the cold waves from Siberia.
  2. Indo-gangetic plain would be devoid of such extensive alluvial soils.
  3. The pattern of monsoon would be different from what it is at present.

Which of the statements given above is/are correct?

Explanation: D. All the statements given in the question are correct.

345. The latitudes that pass through Sikkim also pass through:

Explanation: A. The latitude that passes through Sikkim also pass through Rajasthan.

346. Though coffee and tea both are cultivated on hill slopes, there is some difference between them regarding their cultivation. In this context, consider the following statements:
  1. Coffee plant requires a hot and humid climate of tropical areas whereas tea can be cultivated in both tropical and subtropical areas.
  2. Coffee is propagated by seeds but tea is propagated by stem cuttings only.

Which of the statements given above is/ are correct?

Explanation: C. Both 1 and 2.

347. With reference to soil conservation, consider the following practices:
  1. Crop rotation
  2. Sand fences
  3. Terracing
  4. Wind breaks

Which of the above are considered appropriate methods for soil conservation in India?

Explanation: C. The different types of soil erosion in India are Normal or Geologic Erosion, Accelerated soil Erosion, Wind Erosion, Water Erosion, sheet Erosion, Gully Erosion, Land slides or slip Erosion and stream-bank Erosion.

348. An objective of the National Food Security Mission is to increase the production of certain crops through area expansion and productivity enhancement in a sustainable manner in the identified districts of the country. What are those crops?

Explanation: B. National Food Security Mission launched by government of India to increase the food production of rice, wheat and pulses and commercial gaps like sugarcane jute. The target is to increase the production of rice by 10 million tones, wheat by 8 million tones and pulses, by 2 million tonnes by the end of Eleventh Plan.

349. Due to their extensive rice cultivation, some regions may be contributing to global warming. To what possible reason/reasons is this is attributable?
  1. The anaerobic conditions associated with rice cultivation cause the emission of methane.
  2. When nitrogen based fertilizers are used, nitrous oxide is emitted from the cultivated soil.

Which of the statements given above is/are correct?

Explanation: C. Both 1 and 2.

350. Tamil Nadu is a leading producer of mill-made cotton yarn in the country. What could be the reason?
  1. Tamil Nadu is a leading producer of mill-made cotton yarn in the country. What could be the reason?
  2. Rich pool of skilled labour is available.

Which of the above is/are the correct reasons?

Explanation: B. The predominant soil in Tamil Nadu is red which supports cotton cultivation and a rich pool of skilled labour is available in Tamil Nadu.

351. Given below are the names of four energy crops. Which one of them can be cultivated for ethanol?

Explanation: B. Maize.

352. Between India and East Asia, the navigation time and distance can be greatly reduced by which of the following?
  1. Deepening the Malacca straits between Malaysia and Indonesia.
  2. Opening a new canal across the kra isthmus between the gulf of Siam and Andaman sea.

Which of the statements given above is/are correct?

Explanation: B. Opening a new canal in Kra Isthmus can reduce the navigation time and distance. It

353. The Brahmaputra, Irrawaddy and Mekong rivers originate in Tibet narrow and parallel mountain ranges in their upper reaches. Of these rivers, Brahmaputra makes a "U" turn in its course to flow into India. This "U" turn is due to?

Explanation: B. Brahamputra originates neat Mt. Kailas and is known to take a U turn near Mount Namcha Barwa. This U turn is also known as Great Bend. The U Turn is because of the 180

354. A state in India has the following characteristics:
  1. Its northern part is arid and semiarid.
  2. Its central part produces cotton.
  3. Cultivation of cash crops is predominant over food crops.

Which one of the following states has all of the above characteristics ?

Explanation: B. The North of Gujarat shares its climate with Rajasthan and can be termed as semi-arid and arid type of climate. In agriculture, cash crop is a crop which grown for profit. Sugarcane, cotton, tobacco, oilseeds and peanuts are profitable cash crops. Cash crops are characteristic of the states, agricultural economy.

355. La Nina is suspected to have caused recent floods in Australia. How is La Nina different from EI Nino?
  1. La Nina is characterized by unusually cold ocean temperature in equatorial Indian ocean whereas EI Nino is characterized by unusually warm ocean temperature in the equatorial pacific ocean.
  2. EI Nino has adverse effect on southwest monsoon of India, but La Nina has no effect on monsoon climate.

Which of the statements given above is/are correct?

Explanation: D. La Nina is a coupled ocean atmosphere phenomenon that is the countrrpart of El-Nino as part of the broader El Nio-Southern Oscillation climate pattern. During a period of La-Nina, the sea surface temperature across the equatorial Eastern central pacific ocean will be lower than normal by 3-50C. Some of weather effects of La Nina include abnormally heavy monsoons in India and Southeast Asia, cool and wet winter weather in Southeastern Africa, Wet weather in eastern Australia, cold winter in western Canada and northwestern United States, winter drought in the Southern United States, warm and wet weather in northeastern United States, and an extremely wet winter in Southwestern Canada and north-western USA.

356. With reference to micro irrigation, which of the following statements is/are correct?
  1. Fertilizer/nutrient loss can be reduced.
  2. It is the only means of irrigation in dry land farming.
  3. In some areas of farming, receding of ground water table can be checked.

Select the correct answer using the codes given below:

Explanation: C. The first statement is correct because with Micro irrigation, loss of nutrients can be reduced. The second statement is incorrect. The third statement is correct, because Micro irrigation has evolved from the sub-irrigation where irrigation water is supplied to the plant by "raising the water table" up to the root zone.

357. Salinization occurs when the irrigation water accumulated in the soil evaporates, leaving behind salts and minerals. What are the effects of Salinization on the irrigated land?

Explanation: B. Unless the salts and minerals are washed down into the groundwater, the Sodium and other ions are absorbed by the colloidal clay particles. It leads to the deflocculating of the particles and the soil becomes structure less and impermeable to water.

358. Westerlies in southern hemisphere are stronger and persistent than in northern hemisphere. Why?
  1. Southern hemisphere has less landmass as compared to northern hemisphere.
  2. Coriolis force is higher in southern hemisphere as compared to northern hemisphere.

Which of the statements given above is/are correct?

Explanation: A. The higher speed and greater persistence of the westerlies in the Southern Hemisphere are caused by the difference in the atmospheric pressure patterns as well as its variation from that of the Northern hemisphere. The landmass in the southern hemisphere is comparatively less and average annual pressure decreases much more rapidly on the pole ward side of the high pressure belt. Statement 1 is correct.

359. What could be the main reason/reasons for the formation of African and Eurasian desert belt?
  1. It is located in the subtropical high pressure cells.
  2. It is under the influence of warm ocean currents.

Which of the statements given above is/are correct in this context?

Explanation: A. The Eurasian Belt is related to cold currents not warm current, so statement 2 is incorrect.

360. The jet aircrafts fly very easily and smoothly in the lower stratosphere. What could be the appropriate explanation?
  1. There are no clouds or water vapour in the lower stratosphere.
  2. There are no vertical winds in the lower stratosphere.

Which of the statements given above is/are correct in this context?

Explanation: B. Reason is the low air density that reduces parasitic drag on the airframe. It also allows them to stay above any hard weather (extreme turbulence). Air flow in the stratosphere is much lesser turbulent than in the troposphere and this is the reason that jet aircraft like to cruise at stratospheric altitudes for, the flight is less "bumpy." So statement 2 is correct. Majority of clouds form in the Earth

361. A layer in the Earth's atmosphere called ionosphere facilities radio communication. Why?
  1. The presence of ozone causes the reflection of radio waves to earth.
  2. Radio waves have a very long wavelength.

Which of the statements given above is/are correct?

Explanation: B. Statement 1 is wrong, ionosphere contains charged particles due to which electromagnetic radio waves gets reflected and communication becomes possible. Ozone is found mainly in stratosphere and in traces in troposphere. Its main function is absorption of UV rays. While the ions give the ionosphere its name, it is the free electrons that effect the radio waves and radio communications. Also Radio waves have the longest wave length in the EM spectrum.

362. Which of the following is/are cited by the scientists as evidence/evidences for the continued expansion of universe?
  1. Detection of microwaves in space
  2. Observation of redshift phenomenon in space
  3. Movement of asteroids in space
  4. Occurrence of supernova explosions in space

Select the correct answer using the codes given below:

Explanation: A. The cosmic microwave background (CMB) is the thermal radiation left over from the "Big Bang". The CMB is well explained as radiation left over from an early stage in the development of the universe and its discovery is considered a landmarks test of the Big Bang model of the universe. Red shift and Blueshift describes how light changes as objects in space (such as stars or galaxies) move closer or farther away from us. The concept is key to charting the universe's expansion.

363. Consider the following factors:
  1. Rotation of the Earth
  2. Air pressure and wind
  3. Density of ocean water
  4. Revolution of the Earth

Which of the above factors influence the ocean currents?

Explanation: B. Rotation of the Earth, Air pressure and wind, Density of ocean water. Revolution has no impact on ocean currents.

364. Normally, the temperature decreases with the increase in height from the Earth's surface, because
  1. the atmosphere can be heated upwards only from the Earth's surface
  2. there is more moisture in the upper atmosphere
  3. the air is less dense in the upper atmosphere

Select the correct answer using the codes given below:

Explanation: C. Atmosphere is heated by infrared radiation, Moisture is more in lower atmosphere. In the upper atmosphere Air is less dense which hold less heat thus temperature is low.

365. Which one of the following is the characteristic climate of the Tropical Savannah Region?

Explanation: D. Savannah covers approximately 20% of the Earth's land area. The largest area of Savannah is in Africa. The tropical Savannah region has a definite dry and wet season. Savannah grasslands are much richer in humus than the equatorial forests.

366. A particular State in India has the following characteristics:
  1. It is located on the same latitude which passes through northern Rajasthan.
  2. It has over 80% of its area under forest cover.
  3. Over 12% of forest cover constitutes Protected Area Network in this State.

Which one among the following States has all the above characteristics?

Explanation: A. Arunachal Pradesh.

367. With reference to the wetlands of India, consider the following statements:
  1. The country's total geographical area under the category of wetlands is recorded more in Gujarat as compared to other States.
  2. In India, the total geographical area of coastal wetlands is larger than that of inland wetlands.

Which of the statements given above is/are correct?

Explanation: A. The countries total geographical area under the category of wetlands is more in Gujarat as compared to other states. India has 60% of in land wetland out of total wetland.

368. When you travel in Himalayas, you will see the following:
  1. Deep gorges
  2. U-turn river courses
  3. Parallel mountain ranges
  4. Steep gradients causing land-sliding

Which of the above can be said to be the evidences for Himalayas being young fold mountains?

Explanation: D. All statements are correct.

369. Consider the following statements:
  1. The duration of the monsoon decreases from southern India to northern India.
  2. The amount of annual rainfall in the northern plains of India decreases from east to west.

Which of the statements given above is/are correct?

Explanation: C. Both statements are correct.

370. Which of the following is the chief characteristic of 'mixed farming'?

Explanation: C. Mixed farming refer to rearing of animals and cultivation of crops together.

371. Consider the following crops of India:
  1. Cowpea
  2. Green gram
  3. Pigeon pea

Which of the above is/are used as pulse, fodder and green manure?

Explanation: D. All these crops are leguminous crop (Pulses crops), and have the capacity to fix atmospheric nitrogen through symbiotic nitrogen fixation.

372. Consider the following crops of India:
  1. Groundnut
  2. Sesamum
  3. Pearl millet

Which of the above is/are predominantly rained crop/crops?

Explanation: D. All of them are predominantly rain fed crops.

373. Which one of the following pairs is correctly matched?

Explanation: B. Atlas mountain ranges are situated in the north western part of Africa. They extend almost 2000 km. They pass through Morocco, Algeria and Tunisia.

374. Variations in the length of daytime and nighttime from season to season are due to

Explanation: D. Variations in the length of daytime and nighttion from season to season are due to the revolution of the earth on a tilted axis.

375. On the planet earth, most of the freshwater exists as ice caps and glaciers. Out of the remaining freshwater, the largest proportion

Explanation: C. Out of all the water on Earth, Saline water in oceans, seas and saline groundwater make up about 97% of it. Only 2.5 - 2.75% is fresh water, including 1.75-2% frozen in glaciers, ice and snow, 0.5 - 0.75% as fresh groundwater and soil moisture and less than 0.01% of it as surface water in lakes, Swamps and rivers.

376. Consider the following pairs:
  1. Electromagnetic radiation
  2. Geothermal energy
  3. Gravitational force
  4. Plate movements

Which of the above are responsible for bringing dynamic changes on the surface of the earth?

Explanation: D. From electromagnetic radiation to revolution of the earth, everything is responsible for bringing dynamic changes on the surface of the earth. For example: Electromagnetic radiation brings changes in the field of microwaves, wavelengths of radio, UV rays, infra red rays, X rays and gamma rays. Geothermal energy is the heat received from the earth

377. The most important fishing grounds of the world are found in the regions where

Explanation: C. The mixing of warm and cold current in the region where planktons are found is food for fishes. The temperature is just right for them to survive. The temperature is just right for the growth of fish food called planktons.

378. Which of the following is/are unique characteristic/ characteristics of equatorial forests?
  1. Presence of tall, closely set trees with crowns forming a continuous canopy
  2. Coexistence of a large number of species
  3. Presence of numerous varieties of epiphytes

Select the correct answer using the codes given below.

Explanation: D. The canopy is the primary layer of the forest forming a roof over the two remaining layers. The densest of the biodiversity is found here along with a large variety of epiphytes.

379. The annual range of temperature in the interior of the continents is high as compared to coastal areas. What is/are the reason/reasons?
  1. Thermal difference between land and water
  2. Variation in altitude between continents and oceans
  3. Presence of strong winds in the interior
  4. Heavy rains in the interior as compared to coasts

Select the correct answer using the codes given below.

Explanation: A. The first statement is correct. One major factor affecting the distribution of the temperature of Earth is distribution of Land and Oceans. Since there is more land in Northern Hemisphere and more waters in Southern hemisphere and there is a big difference between the specific heat of land and water; the loss of heat from the continents is bigger than the oceans. The continents get heated faster and get cooled faster in comparison to the Oceans. This is the reason that the temperatures of the Oceans are moderate while that of continents is extreme. The moderating effect on temperature of the land due to proximity of the seas is called Maritime influence. The increasing effect on temperature of the land at interior of the continents is called Continental Influence.

380. "Climate is extreme, rainfall is scanty and the people used to be nomadic herders."

The above statement best describes which of the following regions?

Explanation: B. The central Asian steppes run through Kazakhstan, Turkmenistan, Uzbekistan and Mongolia. The climate here is harsh with dust storms, little to no rainfall and temperature ranging from

381. Consider the following pairs:
  1. Corbett National Park : Ganga
  2. Kaziranga National Park : Manas
  3. Silent Valley: National Park : Kaveri

Which of the above pairs is/are correctly matched?

Explanation: D. Through Corbett National Park Ramganga flows (not Ganga) which is a tributary of Ganges. Through Silent Valley National Park river Bhavani flows which is a tributary of Kaveri. Kaziranga and Manas are both national parks

382. The Narmada river flows to the west, while most other large peninsular rivers flow to the east. Why?
  1. It occupies a linear rift valley.
  2. It flows between the Vindhyas and the Satpuras.
  3. The land slopes to the west from Central India.

Select the correct answer using the codes given below.

Explanation: A. 1 only.

383. Which of the following is/are the characteristic/ characteristics of Indian coal?
  1. High ash content
  2. Low sulphur content
  3. Low ash fusion temperature

Select the correct answer using the codes given below.

Explanation: A. Indian coal has high ash content and low calorific value. It has low sulphur and low phosphorous content but high ash fusion temperature.

384. Which of the following statements regarding laterite soils of India are correct?
  1. They are generally red in colour.
  2. They are rich in nitrogen and potash.
  3. They are well-developed in Rajasthan and UP.
  4. Tapioca and cashew nuts grow well on these soils.

Select the correct answer using the codes given below.

Explanation: C. Laterite soil is rusty red in colour due to iron oxide present in it. In the lateritic soil cashews and tapiocas can be grown.

385. Consider the following statements:
  1. Natural gas occurs in the Gondwana beds.
  2. Mica occurs in abundance in Kodarma.
  3. Dharwars are famous for petroleum.

Which of the statements given above is/are correct?

Explanation: B. Dharwar rocks are non fossilliferous rather they are metalliferous. They bear out gold, iron ore, manganese mica, cobalt, chromium, copper, tungsten, lead, nickel, precious stones and budding stones. Kodarma is a store house of mica and Gondwana beds has coal and not natural gass.

386. Contour bunding is a method of soil conservation used in

Explanation: D. Contour Bunding is one of the simple method of soil and water conservation. This technique is used at places where the land is sloppy. Due to slope, soil and nutrients erode fast which makes agriculture on this land very uneconomical. To adopt this technique the agriculture fields contours are marked and then the bunds are taken along the contours.

387. Consider the following crops:
  1. Cotton
  2. Groundnut
  3. Rice
  4. Wheat

Which of these are Kharif crops?

Explanation: C. Rice is the main kharif crop and groundnut and cotton are also the kharif crops wheat is a rabi.

388. Which of the following phenomena might have influenced the evolution of organisms?
  1. Continental drift
  2. Glacial cycles

Select the correct answer using the code given below.

Explanation: C. Continental Drift has effected the evolution of animals, the world's geographical positions and the world's climates. The split of the original super land mass Pangea into Gondwanaland and the laurasia created new geographical/climatic regions. The rearrangement and displacement of huge landmasses has helped create the diversity which we see in modern day species. The final stages of evolution of Genus Homo occurs in the last 3 glacial cycles.

389. Turkey is located between

Explanation: B. Turkey's smaller part is in Southeastern Europe and its larger part in Western Asia which is located between Black Sea and Mediterranean.

390. Consider the following pairs:
  1. Chechnya : Russian Federation
  2. Darfur : Mali
  3. Swat Valley : Iraq

Which of the above pairs is/are correctly matched?

Explanation: A. The Chechen Republic is a federal subject (a republic) of Russia. Darfur is a region in western Sudan. Swat is a valley and an administrative district in Pakistan.

391. What is the correct sequence of occurrence of the following cities in South-East Asia as one proceeds from south to north?
  1. Bangkok
  2. Hanoi
  3. Jakarta
  4. Singapore

Select the correct answer using the code given below.

Explanation: C. The correct sequence of occurrence of the following cities in South-East Asia as one proceeds from south to north is Jakarta-Singapore-Bangkok and Hanoi.

392. Consider the following towns of India:

Explanation: B. Chanderi in Madhyapradesh and Kancheepuram in Tamil Nadu are famous for Silk sarees.

393. Which of the above are famous for the production of traditional sarees/ fabric?

Explanation: B. Chanderi in Madhyapradesh and Kancheepuram in Tamil Nadu are famous for Silk sarees.

394. Consider the following pairs:
  1. NH 4 Chennai and Hyderabad
  2. NH 6 Mumbai and Kolkata
  3. NH 15 Ahmedabad and Jodhpur

Which of the above pairs is/are correctly matched?

Explanation: D. NH 4 is a major National Highway in Western and Southern India. It connects Mumbai with Chennai. NH 6, runs through Gujarat, Maharashtra, Chhattisgarh, Odisha, Jharkhand and West Bengal state in India. It connects Surat with Kolkata. NH 15 is a major National Highway in Western and Northwestern India. NH 15 connects Samakhiali in Gujarat with Pathankot in Punjab.

395. The seasonal reversal of winds is the typical characteristic of

Explanation: C. Monsoon is the only climate which is having reversal of wind between seasons.

396. Consider the following rivers:
  1. Barak
  2. Lohit
  3. Subansiri

Which of the above flows/flow through Arunachal Pradesh?

Explanation: B. Rivers Lohit and Subansiri flow through Arunachal Pradesh. River Barak flows in south Assam and Manipur. River lohit and subanseri flows through Arunachal Pradesh. Barak river flows in south Assam and Manipur.

397. Consider the following pairs:
  1. Harike Wetlands : Confluence of Beas and Satluj
  2. Keoladeo Ghana National Park : Confluence of Banas and Chambal
  3. Kolleru Lake : Confluence of Musi and Krishna

Which of the above pairs is/ are correctly matched?

Explanation: A. Harike Wetlands is at Confluence of Beas and Satluj/Sutlej. The Keoladeo National Park formerly known as the Bharatpur Bird Sanctuary in Bharatpur is at the confluence of two rivers, the Gambhir and Banganga. Kolleru Lake is one of the largest freshwater lakes in India located in state of Andhra Pradesh. Kolleru is located between Krishna and Godavari delta.

398. Consider the following pairs:
  1. Cardamom Hills : Coromandel Coast
  2. Kaimur Hills : Konkan Coast
  3. Mahadeo Hills : Central India
  4. Mikir Hills : North-East India

Which of the above pairs are correctly matched?

Explanation: C. The Cardamom Hills are southern hills of India and part of the southern Western Ghats located in southeast Kerala and southwest Tamil Nadu. They are not in coromandel coast. Kaimur Range is the eastern portion of the Vindhya Range extending from Madhya Pradesh to Bihar. They are not in konkan coast. The Mahadeo Hills are in Madhya Pradesh, state of central India.Mikir hills are in assam i.e. in North East India.

399. If you travel through the Himalayas, you are likely to see which of the following plants naturally growing there?
  1. Oak
  2. Rhododendron
  3. Sandalwood

Select the correct answer using the code given below

Explanation: A. Oak and Rhododendron is found in Himalayan region while sandalwood is found in South India.

400. With reference to 'Changpa' community of India, consider the following statements:
  1. They live mainly in the State of Uttarakhand.
  2. They rear the Pashmina goats that yield a fine wool
  3. They are kept in the category of Scheduled Tribes.

Which of the statements given above is/are correct?

Explanation: B. The Changpa are a semi-nomadic Tibetan ethnic group found mainly in Zanskar region of Jammu and Kashmir. They rear the Pashmina goats that yield fine wool. They are kept in the category of Scheduled Tribes.

401. With reference to Neem tree, consider the following statements:
  1. Neem oil can be used as a pesticide to control the proliferation of some species of insects and mites
  2. Neem seeds are used in the manufacture of biofuels and hospital detergents.
  3. Neem oil has applications in pharmaceutical industry.

Which of the statements given above is/are correct?

Explanation: C. Correct option is C.

402. Consider the following pairs:
  1. Kinnaur : Areca nut
  2. Mewat : Mango
  3. Coromandel : Soya bean

Which of the above pairs is/ are correctly matched?

Explanation: D. Kinnaur (Himachal) is famous for apples. Areca nut mostly confined to Karnataka, Kerala and Assam. Mewat in haryana is not famous for mango. UP is famous for mango. Similarly, coromandel coast is not famous for soya beans.Top two producers of soya are Madhya Pradesh and Maharashtra which are outside coromandel coast. Therefore, (D) is the correct option.

403. Consider the following techniques phenomena:
  1. Budding and grafting in fruit plants
  2. Cytoplasmic male sterility
  3. Gene silencing

Which of the above is/are used to create transgenic crops?

Explanation: B. Correct option is B.

404. Consider the following statements:
  1. Maize can be used for the production of starch.
  2. Oil extracted from maize can be a feedstock for biodiesel.
  3. Alcoholic beverages can be produced by using maize.

Which of the statements given above is/are correct?

Explanation: D. All three statements are correct as Maize can be used for the production of starch.Oil extracted from maize can be a feedstock for biodiesel. Alcoholic beverages can be produced by using maize.

405. What are the significances of a practical approach to sugarcane production known as 'Sustainable Sugarcane Initiative'?
  1. Seed cost is very low in this compared to the conventional method of cultivation.
  2. Drip irrigation can be practiced very effectively in this.
  3. There is no application of chemical/inorganic fertilizers at all in this.
  4. The scope for intercropping is more in this compared to the conventional method of cultivation.

Select the correct answer using the code given below.

Explanation: B. Correct option is B.

406. Consider the following pairs:
  1. Drought-Prone Area Programme : Ministry of Agriculture
  2. Desert Development Programme and Forests : Ministry of Environment
  3. National Watershed Project Development for Rainfed Areas: Ministry of Rural Development

Which of the above pairs is/ are correctly matched?

Explanation: D. The draught prone area programme is by ministry of Rural development to minimise the adverse effects of drought on production of crops and livestock and productivity of land, water and human resources. The desert development programme is also by ministry of Rural development to minimise the adverse effect of drought and control desertification. National Watershed Development Project for Rainfed Areas is by ministry of agriculture.

407. What are the benefits of implementing the 'Integrated Watershed Development Programme'?
  1. Prevention of soil runoff
  2. Linking the country's perennial rivers with seasonal rivers
  3. Rainwater harvesting and recharge of groundwater table
  4. Regeneration of natural vegetation

Select the correct answer using the code given below

Explanation: C. Integrated Watershed Management Programme (IWMP) was launched during 2009-10.It aims at Prevention of soil runoff, Rainwater harvesting and recharge of groundwater table and Regeneration of natural vegetation.

408. In India, cluster bean (Guar) is traditionally used as a vegetable or animal feed, but recently the cultivation of this has assumed significance. Which one of the following statements is correct in this context?

Explanation: B. Guar gum is used in hydraulic fracturing technology during shale gas extraction.

409. Which of the following statements is/are correct regarding vegetative propagation of plants?
  1. Vegetative propagation produces clonal population.
  2. Vegetative propagation helps in eliminating the virus
  3. Vegetative propagation can be practiced most of the year

Select the correct answer using the code given below.

Explanation: C. Vegetative reproduction is a form of asexual reproduction in plants. It is a process by which new organisms arise without production of seeds .It helps in development of clones. Vegetative propagation involves only mitosis, this ensures that the genetic information in DNA of vegetative progeny (child) is same as in the mother plant and can be practiced throughout the year. However it does not helps in elimination of viruses. Plant once systematically infected with a virus, usually remains infected for its lifetime. Thus any vegetative parts taken for propagation remains infected.

410. Consider the following statements
  1. The winds which blow between 30
  2. The moist air masses that cause winter rains in North-Western region of India are part of westerlies.

Which of the statements given above is/are correct?

Explanation: C. The Westerlies are prevailing winds from the west toward the east in the middle latitudes between 30 and 60 degrees latitude. They originate from the high-pressure areas in the horse latitudes and tend towards the poles and steer extra tropical cyclones in this general manner. The moist air masses that cause winter rains in North-Western region of India are part of westerlies.

411. Tides occur in the oceans and seas due to which among the following?
  1. Gravitational force of the Sun
  2. Gravitational force of the Moon
  3. Centrifugal force of the Earth

Select the correct answer using the code given below.

Explanation: D. Tides are the rise and fall of sea levels caused by the combined effects of gravitational forces exerted by the Moon, Sun, and rotation of the Earth.

412. "Each day is more or less the same, the morning is clear and bright with a sea breeze; as the Sun climbs high in the sky, heat mounts up, dark clouds form, then rain comes with thunder and lightning. But rain is soon over." Which of the following regions is described in the above passage?

Explanation: B. The passage points out equatorial region.

413. In the South Atlantic and South-Eastern Pacific regions in tropical latitudes, cyclone does not originate. What is the reason?

Explanation: A. Correct option is A.

414. What explains the eastward flow of the equatorial counter-current?

Explanation: A. The Earth's rotation on its axis explains the eastward flow of the equatorial counter-current. If the earth would have been rotating east to west, the piled up water would have come down on the west side. Therefore, essentially it's the earth's rotation that explains the eastward flow of equatorial counter current.

415. Which one of the following countries of South-West Asia does not open out to the Mediterranean Sea?

Explanation: B. Jordan does not open out to the Mediterranean Sea.

416. In a particular region in India, the local people train the roots of living trees into robust bridges across the streams. As the time passes, these bridges become stronger. These unique 'living root bridges' are found in

Explanation: A. In the present-day Meghalaya state of northeast India is a form of tree shaping, which creates these suspension bridges, they are handmade from the aerial roots of living banyan fig trees.

417. Consider the following States
  1. Arunachal Pradesh
  2. Himachal Pradesh
  3. Mizoram

In which of the above States do 'Tropical Wet Evergreen Forests' occur?

Explanation: C. Tropical Wet Evergreen Forests are found in Assam, Arunachal Pradesh, Meghalaya, Nagaland, Tripura, West Bengal and Andaman and Nicobar Island and on the eastern and western slopes of the Western Ghats in such states as Tamil Nadu, Karnataka, Kerala and Maharashtra.

418. Which one of the following pairs of States of India indicates the eastern most and western most State?

Explanation: D. Arunachal Pradesh and Gujarat are the easternmost and westernmost States.

419. Consider the following rivers:
  1. Vamsadhara
  2. Indravati
  3. Pranahita
  4. Pennar

Which of the above are tributaries of Godavari?

Explanation: D. Major tributaries of the river include the Purna (South), Pravara, Indravati, Manjira River, Bindusara River, Sabari River, Wainganga, and Wardha River. Pranahita is the name given to the combined flow of the rivers Wardha and Wainganga.

420. Which one of the following regions of India has a combination of mangrove forest, evergreen forest and deciduous forest?

Explanation: D. Andaman and Nicobar Islands is rich in biodiversity. Different types of forests as Tropical Evergreen forests, Moist Deciduous forests, Mangrove forests, Littoral forests are found here.

421. Which of the following has/have been accorded 'Geographical Indication' status?
  1. Banaras Brocades and Sarees
  2. Rajasthani Daal-Bati-Churma
  3. Tirupathi Laddu

Select the correct answer using the code given below.

Explanation: C. Banaras Brocades and Sarees (Logo) has been accorded 'Geographical Indication' status. Rajasthani Daal-Bati-Churma is not in the list of Geographical Indications in Indus - Therefore option 'C' is correct.

422. The substitution of steel for wooden ploughs in agricultural production is an example of

Explanation: B. Capital-augmenting technological progress results in the more productive use of existing capital goods; for example, the substitution of steel from wooden ploughs in agricultural production.

423. Which one of the following best describes the main objective of 'Seed Village Concept'?

Explanation: B. Seed village concept is to promote the quality seed production of foundation and certified seed classes. The area which is suitable for raising a particular crop will be selected, and raised with single variety of a kind. Suitable area for seed production will be identified by the Scientists. The foundation/ certified seeds or University labelled seeds will be supplied by the University through Krishi Vigyan Kendras (KVKs) and Research Stations at 50% subsidy cost to the identified farmers in the area. The farmers will use these quality seeds and take up their own seed production in a small area (1 acre) for their own use. The crops are Rice, Pulses and Oilseeds.

424. What can be the impact of excessive / inappropriate use of nitrogenous fertilizers in agriculture?
  1. Proliferation of nitrogen-fixing microorganisms in soil can occur.
  2. Increase in the acidity of soil can take place.
  3. Leaching of nitrate to the ground-water can occur.

Select the correct answer using the code given below.

Explanation: C. Excessive / inappropriate use of nitrogenous fertilizers increase the acidity of soil and Leaching of nitrate to the ground water.

425. In India, markets in agricultural products are regulated under the

Explanation: B. Agricultural Produce Market Committee Acts of respective states are responsible for markets in agricultural products.

426. The terms 'Agreement on Agriculture', 'Agreement on the Application of Sanitary and Phytosanitary Measures' and 'Peace Clause' appear in the news frequently in the context of the affairs of the

Explanation: C. Agreement on Agriculture', 'Agreement on the Application of Sanitary and Phytosanitary Measures' and 'Peace Clause' are related to World Trade Organisation.

427. Which of the following is/are tributary tributaries of Brahmaputra?
  1. Dibang
  2. Kameng
  3. Lohit

Select the correct answer using the code given below.

Explanation: D. The Brahmaputra enters India in the state of Arunachal Pradesh from its original source Tibet, and is joined by the Dibang River and the Lohit River at the head of the Assam Valley. It is joined in Sonitpur by the Kameng River (or Jia Bhoreli). Brahmaputra's main left bank tributaries, viz., Dibang or Sikang and Lohit. The important right bank tributaries are the Subansiri, Kameng, Manas and Sankosh.

428. In which of the following regions of India are shale gas resources found?
  1. Cambay Basin
  2. Cauvery Basin
  3. Krishna-Godavari Basin

Select the correct answer using the code given below.

Explanation: D. Research is under process for presence of shale gas in Cambay basin at Mehsana, Ahmedabad and Bharuch districts of Gujarat, Cauvery basin at Nagapattinam in Tamil Nadu and in KG Basin at East and West Godavari districts of Andhra Pradesh.

429. The FAO accords the status of 'Globally Important Agricultural Heritage System (GIAHS)' to traditional agricultural systems. What is the overall goal of this initiative?
  1. To provide modern technology, training in modern farming methods and financial support to local communities of identified GIAHS so as to greatly enhance their agricultural productivity
  2. To identify and safeguard eco-friendly traditional farm practices and their associated landscapers, agricultural biodiversity and knowledge systems of the local communities
  3. To provide Geographical Indication status to all the varieties of agricultural produce in such identified GIAHS

Select the correct answer using the code given below

Explanation: B. In order to safeguard and support the world's agri-cultural heritage systems, in 2002 FAO started an initiative for the dynamic conservation of Globally Important Agricultural Heritage Systems (GIAHS). The GIAHS Initiative promotes public understanding, awareness, national and international recognition of Agricultural Heritage systems. It aims to identify and ensure global recognition of the importance of unique traditional agricultural systems for food security and sustainable development. The GIAHS initiative explicitly recognises that change in "traditional" political, social and economic processes is inevitable; they cannot be frozen or re-created. There is no provision to give Geographical indication to

430. With reference to 'Initiative for Nutritional Security through Intensive Millets Promotion', which of the following statements is/are correct?
  1. This initiative aims to demonstrate the improved production and post-harvest technologies, and to demonstrate value addition techniques, in an integrated manner, with cluster approach.
  2. Poor, small, marginal and tribal farmers have larger stake in this scheme.
  3. An important objective of the scheme is to encourage farmers of commercial crops to shift to millet cultivation by offering them free kits of critical inputs of nutrients and micro irrigation equipment.

Select the correct answer using the code given below.

Explanation: C. This initiative aims to demonstrate the improved production and post-harvest technologies in an integrated manner to catalyze increased production of millets in the country. The scheme, through processing and value addition techniques, is expected to generate consumer demand for millet based food products. Poor, small, marginal and tribal farmers have larger stake in this scheme. 1st and 2nd statements are right. Policy aims to give free kits but "wanting to shift commercial crop farmers to millet" doesn't fit logic. There is no such mention in the scheme, nor any talk about giving micro irrigation equipment. Hence, 3rd statement wrong.

431. Which of the following is/are the advantage / advantages of practising drip irrigation?
  1. Reduction in weed
  2. Reduction in soil salinity
  3. Reduction in soil erosion

Select the correct answer using the code given below

Explanation: C. Correct option is C.

432. Why does the Government of India promote the use of Neem-coated Urea' in agriculture?

Explanation: B. Spraying urea with neem oil slows the release of nitrogen, by about 10 to 15 per cent, concomitantly reducing consumption of the fertiliser. Many research studies in India have conclusively established that neem oil acts as an effective nitrification inhibitor if coated onto urea. Spraying urea with neem oil slows the release of nitrogen, by about 10 to 15 per cent, concomitantly reducing consumption of the fertiliser.

433. With reference to 'Indian Ocean Dipole (IOD)' sometimes mentioned in the news while forecasting Indian monsoon, which of the following statements is/are correct?
  1. IOD phenomenon is characterized by a difference in sea surface temperature between tropical Western Indian Ocean and tropical Eastern Pacific Ocean.
  2. An IOD phenomenon can influence an El Nino's impact on the monsoon.

Select the correct answer using the code given below:

Explanation: B. Indian Ocean Dipole (IOD) is an event which the western Indian Ocean becomes alternately warmer and then colder than the eastern part of the Indian ocean. A positive IOD leads to greater monsoon rainfall and more active (above normal rainfall) monsoon days while negative IOD leads to less rainfall and more monsoon break days (no rainfall). So yes, IOD can influence El Nino's impact on Monsoon.

434. Which of the following is geographically closest to Great Nicobar?

Explanation: A. Sumatra.

435. Mediterranean Sea is a border of which of the following countries?
  1. Jordan
  2. Iraq
  3. Lebanon
  4. Syria

Select the correct answer using the code given below:

Explanation: C. Mediterranean Sea doesn't touch Jordan and Iraq.

436. At one of the places in India, if you stand on the seashore and watch the sea, you will find that the sea water recedes from the shore line a few kilometres and comes back to the shore, twice a day, and you can actually walk on the sea floor when the water recedes. This unique phenomenon is seen at

Explanation: C. Chandipur beach has a unique distinction on its own. Unlike other beaches, the sea water here recedes away from the shore line about five km twice a day, an unusual phenomenon, rarely found anywhere.

437. Consider the following statements: The nation-wide 'Soil Health Card Scheme' aims at
  1. expanding the cultivable area under irrigation.
  2. enabling the banks to assess the quantum of loans to be granted to farmers on the basis of soil quality.
  3. checking the overuse of fertilizers in farmlands.

Which of the above statements is/are correct?

Explanation: B. A soil health card provides information about 12 soil parameters, so farmer can use appropriate fertilizers.

438. What is/are the advantage/advantages of implementing the 'National Agriculture Market' (NAM) scheme?
  1. It is a pan-India electronic trading portal for agricultural commodities.
  2. It provides the farmers access to nationwide market, with prices commensurate with the quality of their produce.

Select the correct answer using the code given below:

Explanation: C. Correct option is C.

439. Which of the following practices can help in water conservation in agriculture?
  1. Reduced or zero tillage of the land
  2. Applying gypsum before irrigating the field
  3. Allowing crop residue to remain in the field

Select the correct answer using the code given below:

Explanation: D. Crop residues or other organic matter left in or added to the field improve water penetration and moisture retention. Gypsum application can solve the water percolation process.

440. With reference to agriculture in India, how can the technique of 'genome sequencing', often seen in the news, be used in the immediate future?
  1. Genome sequencing can be used to identify genetic markers for disease resistance and drought tolerance in various crop plants.
  2. This technique helps in reducing the time required to develop new varieties of crop plants
  3. It can be used to decipher the host-pathogen relationships in crops.

Select the correct answer using the code given below:

Explanation: D. Correct option is D.

441. If you travel by road from Kohima to Kottayam, what is the minimum number of States within India through which you can travel, including the origin and the destination?

Explanation: B. Total seven states.

442. Consider the following statements:
  1. In India, the Himalayas are spread over five States only.
  2. Western Ghats are spread over five States only.
  3. Pulicat Lake is spread over two States only.

Which of the statements given above is/are correct?

Explanation: B. Western Ghats are spread over six States: Gujarat ( southern part), Maharastra, Goa, Karnataka, Kerala and Tamilnadu. Himalaya ought to be passing through international border states only i.e. J&K, Himachal, Uttarakhand, Sikkim and Arunanchal Pradesh.

443. With reference to river Teesta, consider the following statements:
  1. The source of river Teesta is the same as that of Brahmaputra but it flows through Sikkim.
  2. River Rangeet originates in Sikkim and it is a tributary of river Teesta.
  3. River Teesta flows into Bay of Bengal on the border of India and Bangladesh.

Which of the statements given above is/are correct?

Explanation: B. 2 only.

444. Consider the following statements
  1. Most of the world's coral reefs are in tropical waters.
  2. More than one-third of the world's coral reefs are located in the territories of Australia, Indonesia and Philippines.
  3. Coral reefs host far more number of animal phyla than those hosted by tropical rainforests.

Which of the statements given above is/are correct?

Explanation: C. Coral reefs are found in tropical and subtropical regions. Most of them are located between Tropics of Capricorn and Cancer. According to UNEP, percentage of world area covered by each region: Indonesia 18%, Australia: 17% and Philippines: 9% so collectively that'll be more than 33%. Coral reefs include a wide range of diversity with 32 of the 34 animal phyla present, in contrast to only 9 phyla represented in tropical rainforest.

445. Which of the following has/have shrunk immensely/ dried up in the recent past due to human activities?
  1. Aral Sea
  2. Black Sea
  3. Lake Baikal

Select the correct answer using the code given below :

Explanation: A. The Aral Sea, in Central Asia, used to be the fourth largest lake in the world, after the Caspian Sea, and Lakes Superior and Victoria Now barely 10% of it is left. Lake Baikal's has been crippled by a series of detrimental phenomena. They include the disappearance of the omul fish, rapid growth of putrid algae and the death of endemic species of sponges across its vast 3.2 million-hectare area.

446. With reference to the circumstances in Indian agriculture, the concept of "Conservation Agriculture" assumes significance. Which of the following fall under the Conservation Agriculture?
  1. Avoiding the monoculture practices
  2. Adopting minimum tillage
  3. Using crop residues to cover soil surface
  4. Adopting spatial and temporal crop sequencing/ crop rotations

Select the correct answer using the code given below:

Explanation: C. minimum soil disturbance by adopting no tillage or minimum tillage and reduced traffic for agricultural operations. maximum soil cover by leaving crop residues on the soil surface. adopt spatial and temporal crop sequencing/crop rotation.

447. With reference to agricultural soils, consider the following statements :
  1. A high content of organic matter in soil drastically reduces its water holding capacity.
  2. Soil does not play any role in the Sulphur cycle.
  3. Irrigation over a period of time can contribute to the salinization of some agricultural lands.

Which of the statements given above is/are correct?

Explanation: B. The process of decay, added by bacterial action, transforms organic matter into humus. Humus enhances water retention capacity of the soil.

448. Which one of the following is an artificial lake?

Explanation: A. Kodaikanal has 154-year-old, artificially built lake. Kolleru Lake Situated in Andhra Pradesh, it is the largest fresh water lake of India It is located between the deltas of the Krishna and Godavari rivers in the Krishna and Godavari districts. The lake serves as a natural flood-balancing reservoir for the two rivers, it's also a Ramsar convention site. Nainital is situated in the foothills of the Himalayas, with the pear shaped Naini lake at its centre. Renuka Lake Situated in the Siarmaur district of Himachal Pradesh, this lake has been named after the goddess Renuka. A lion safari and a zoo are major attractions at Renuka. It is a site for the annual fair in the month of November.

449. Consider the following statements:
  1. The Barren Island volcano is an active volcano located in the Indian territory.
  2. Barren Island lies about 140 km east of Great Nicobar.
  3. The last time the Barren Island volcano erupted was in 1991 and it has remained inactive since then.

Which of the statements given above is/are correct?

Explanation: A. Barren Island volcano became active again in March, 1991. A second phase of eruptions started in 1995.

450. Consider the following statements:
  1. In India, State Governments do not have the power to auction non-coal mines.
  2. Andhra Pradesh and Jharkhand do not have gold mines.
  3. Rajasthan has iron ore mines.

Which of the statements given above is/are correct?

Explanation: D. As per the MMDR Amendment Act 2015 the state governments will conduct auction for grant of mineral concessions. The role of the central government is to prescribe the terms and conditions and procedures subject to which the auction shall be conducted. Largest gold ore (primary) are located in Bihar (44%) followed by Rajasthan (25%) and Karnataka (21%), West Bengal, and Andhra Pradesh (3% each). Hematite and magnetite are the most important iron ores in India. Resources of hematite are spread in Andhra Pradesh, Assam, Bihar, Maharashtra, Madhya Pradesh, Meghalaya, Rajasthan and Uttar Pradesh.

451. Among the following cities, which one lies on a longitude closest to that of Delhi?

Explanation: A. Bengaluru.

452. Which one of the following National Parks lies completely in the temperate alpine zone?

Explanation: D. The Valley of Flowers national park located in the Chamoli district of Uttarakhand and is known for its meadows of endemic alpine flowers and the variety of flora. Situated approximately 3255m above sea level, this splendid park is spread across 87.5 sq km of land. The valley has three sub-alpine between 3,200m and 3,500m which is the limit for trees, lower alpine between 3,500m and 3,700m, and higher alpine above 3,700m. The rich diversity of species reflects the valley's location within a transition zone between the Zanskar and Great Himalayas ranges to the north and south, respectively, and between the Eastern Himalaya and Western Himalaya flora.

453. On 21 June, the Sun

Explanation: A. 21st June is the longest day of the year as countries lying to the north of the equator, including India, witness 'Summer Solstice'. The 'summer solstice' occurs exactly when the earth's axial tilt is most inclined towards the sun at its maximum of 23 degrees 26'.During 'summer solstice', the Northern Hemisphere witnesses its longest day of the year while the Southern Hemisphere sees its shortest day.

454. Consider the following pairs:
  1. Pandharpur Chandrabhaga
  2. Tiruchirappalli Cauvery
  3. Hampi Malaprabha

Which of the pairs given above are correctly matched?

Explanation: A. Pandharpur is a holy place of Shri.Vitthal and Shri.Rukmini. It is also known as the Southern Kashi of India and Kuldaivat of Maharashtra State. The Chandrabhaga (Bhima) river flows through the City.Tiruchirappalli, situated on the banks of the river Cauvery is the fourth largest city in Tamil Nadu. It was a citadel of the early Cholas which later fell to the Pallavas. Hampi is a UNESCO World Heritage Site in India located near Hospet town in the Karnataka state, on south bank of Tungabhadra river.

455. In the context of which of the following do some scientists suggest the use of cirrus cloud thinning technique and the injection of sulphate aerosol into stratosphere?

Explanation: D. Geo-engineering approach to reversing global warming

456. Consider the following States:
  1. Chhattisgarh
  2. Madhya Pradesh
  3. Maharashtra
  4. Odisha

With reference to the States mentioned above, in terms of percentage of forest cover to the total area of State, which one of the following is the correct ascending order?

Explanation: C. 3, 2, 4 & 1

457. Which of the following statements are correct about the deposits of 'methane hydrate'?
  1. Global warming might trigger the release of methane gas from these deposits.
  2. Large deposits of 'methane hydrate' are found in Arctic Tundra and under the seafloor.
  3. Methane in atmosphere oxidizes to carbon dioxide after a decade or two.

Select the correct answer using the code given below.

Explanation: D. 1, 2 and 3.

458. Consider the following:
  1. Carbon monoxide
  2. Methane
  3. Ozone
  4. Sulphur dioxide

Which of the above are released into atmosphere due to the burning of crop/biomass residue?

Explanation: D. 1, 2, 3 and 4.

459. Consider the following pairs:
  1. Adriatic Sea Albania
  2. Black Sea Croatia
  3. Caspian Sea Kazakhstan
  4. Mediterranean Sea Morocco

Which of the pairs given above are correctly matched?

Explanation: B. 1, 3 and 4. The Adriatic Sea is a part of the Mediterranean Sea positioned between the eastern coastline of Italy, and countries of the Balkan Peninsula, from Slovenia, South through Croatia, Montenegro, and to Albania. The Black Sea is an inland sea located between far-southeastern Europe and the far-western edges of the continent of Asia and the country of Turkey. It's bordered by Turkey, and by Bulgaria, Romania, Ukraine, Russia and Georgia. Caspian Sea is an enclosed body of water between Asia and Europe. It is bordered by Iran, Turkmenistan, Kazakhstan, Azerbaijan, Russia. Morocco

460. Consider the following pairs
  1. Bandarpunch Yamuna
  2. Bara Shigri Chenab
  3. Milam Mandakini
  4. Siachen Nubra

Which of the pairs correctly matched?

Explanation: A. An important glacier of Yamuna river basin is Bandarpunch Glacier in the Garhwal division of the Himalayas. It is 12 km long situated on the Northern slopes of Bandarpunch West, Khatling peak and Bandarpunch peak the glacier is formed by three cirque glaciers and then join the river of Yamuna. Bara Shigri is the largest glacier located in Lahaul Spiti region in Chandra Valley, Himachal Pradesh. It is a 30-km long glacier, the second longest glacier in the Himalayas after Gangotri. It flows northwards and feeds the Chenab river. Milam Glacier in Munsiyari, Pithoragarh district, Uttarakhand is the source of Gori Ganga River and not of Mandakini River. Gori Ganga is also an important tributary of Kali River. At an altitude of roughly 5,400 meters (17,700 feet), the Siachen Glacier in Kashmir is a forbidding place. At much lower altitudes, the glacier

461. What is common to the places known as Aliyar, Isapur and Kangsabati?

Explanation: D. Aliyar (Tamil Nadu) Isapur (Maharashtra,) and Kangsabati ( West Bengal) are water reservoirs, where water level reached much below the normal capacity.

462. Why are dewdrops not formed on a cloudy night?

Explanation: B. Clouds reflect back the Earth's radiation.

463. With reference to the cultivation of Khalif crops in India in the last five years, consider the following statements:
  1. Area under rice cultivation is the highest.
  2. Area under the cultivation of jowar is more than that of oilseeds.
  3. Area of cotton cultivation is more than that of sugarcane.
  4. Area under sugarcane cultivation has steadily decreased.

Which of the statements given above are correct?

Explanation: A. 1 and 3 only.

Question No.
Toppers

Top Popular Courses


Newsletter Subscription
SMS Alerts

Important Links

UPSC GS Mains Crash Course - RAW Prelims Answer Key 2024